OAS Prelim 2022-23 Subject-wise Questions & Answers.

Compilation has been done according to the Syllabus mentioned in the OAS Prelim Syllabus.
Compilation has been done according to the Syllabus mentioned in the OAS Prelim Syllabus.


    OAS Prelim Syllabus


    Paper-1 General studies.

    • Current events of national and international importance.

    • History of India and Indian national movements.
    • History of Odisha and Odia Nationalism

    • Indian & Odisha Geography – Physical, Social, Economic Geography of India & the World.

    • Indian polity and governance- Constitution, political system, Panchayati Raj, public policy, Rights Issues, etc.

    • Economic and Social development-sustainable development, poverty, inclusion, Demographics, Social Sector initiatives, etc.

    • General issues on Environmental Ecology, Biodiversity, and climate change that don't require subject specialization.

    • General Science



    Current events of national and international importance.

  1. Which of the following statement/s is/are not correct in respect of G20 Developing Nations? 
  2. (1) It is a block of developing nations established on April 30, 2001

     (2) This group emerged at the 5th Ministerial NATO Conference 

    (3) The Headquarters of G20 developing Nations is at Patna 

    (4) The Member State of G20 Developing Nations is 20 countries 

    (A) 2 and 3 

    (B) 3 and 4 
    (C) 1 and 4 
    (D) 1 and 2  

    Answer: D 

    Explanation: Statements 1 and 2 are incorrect:  The G20 developing nations, also known as G20+, G21, G23, or G20- developing nations, is a coalition of developing countries established on 20 August 2003 during the 5th Ministerial WTO Conference in Cancún, Mexico. Statement 3 is correct and Statement 4 is incorrect: As per the Options, the best possible Answer for Q1. will be “D” 


    https://en.wikipedia.org/wiki/G20_developing_nations#:~:text=The%20G20%20developing%20nations%20(and,September%20to%2014%20September%202003.


    1. During his visit to Australia PM Modi spelled about 3 Cs, 3 Ds, and 3 Es of India Australia friendship. Which one of the following is correct in respect of 3 Cs? 

    (A) Commonwealth, Cricket and Curry 

    (B) Cricket, Career and Candid 
    (C) Cricket, Coffee and Comfort 
    (D) Care, Career and Connect

    Answer: A Explanation: Prime Minister Narendra Modi's speech at the Qudos Bank Arena in Sydney in May 2023. In his speech, PM Modi spoke about the strong and growing ties between India and Australia, which he said were based on three sets of factors: 

    ● The 3 Cs: commonwealth, cricket, and curry
    ● The 3 Ds: diaspora, democracy, and Dosti
    ● The 3 Es: economy, energy, and education

    https://www.livemint.com/news/india/pm-modi-and-es-for-india-australia-11684830813027.html


    1. Which of the following statement/s is/are correct in respect of ‘Sengol’?

    (1) ‘Sengol’ signifies the transfer of power from British to Indians

    (2) It symbolizes the Marathi culture of transfer of power from one King of other King

    (3) The ‘Sengol’ has been reintroduced in the new Parliament Bhavan of India

    (4) Pandit Jawaharlal Nehru accepted ‘Sengol’ at 10.45 PM on August 14, 1947 from Adhinam of Kerala

    (a) 1 and 3

    (b) 2 and 4

    (c) 3 and 4

    (d) 2 and 3


    Answer: A 

    Explanation: 

    • unchecked

      Statement 1 is correct and Statement 4 is incorrect: The Sengol is a sacred symbol of fair and equitable governance that was presented to Jawaharlal Nehru, the first Prime Minister of India, by emissaries from the Thiruvaduthurai Adheenam Matha, a Hindu monastery in Tamil Nadu, on the night of August 14, 1947, to mark the transfer of power from the British to India. 

    • unchecked

      Statement 2 is incorrect: The Sengol, a gold-plated silver scepter with a Nandi, the bull of Lord Shiva, carved at the top, is a sacred symbol of fair and equitable governance in Tamil culture. It is also seen as a symbol of the transfer of power from one thing to another. In Tamil Nadu, the Sengol is traditionally used in religious ceremonies to mark the transfer of power from one hereditary priest to another. It is also used in other ceremonies to mark the transfer of power or authority, such as the inauguration of a new king or the installation of a new government. 

    • unchecked

      Statement 3 is correct: Prime Minister Narendra Modi placed the historic 'Sengol' near the Lok Sabha Speaker's chair in the newly built Parliament building on May 28, 2023, the occasion of the building's inauguration. 


    https://pib.gov.in/PressReleasePage.aspx?PRID=%201926883


    1. Which of the following statements is correct with respect to the withdrawal of Rs. 2000 Currency Notes from circulation?

    (1) This move is a part of RBI’s Compound Note Policy

    (2) Its aim is to provide the Public with high-quality Currency Notes

    (3) Rs. 2000 Currency Notes were introduced in the year 2015

    (4) Printing of Rs. 2000 Currency Notes was stopped in the year 2018-19

    (a) 2 and 4

    (b) 1 and 3

    (c) 3 and 4

    (d) 1 and 4


    Answer: A 

    Explanation: 

    • unchecked

      The government, in consultation with the Reserve Bank of India (RBI), decides the printing of banknotes of specific denominations. In this case, no fresh indent has been placed for printing Rs 2000 denomination notes since 2018-19. 

    • unchecked

      Governments often decide on a currency mix strategy based on economic needs, transactional demands, and attempts to curb issues like black money, terror funding, and hoarding. The shift away from Rs 2000 notes could be part of such a strategy. 

    • unchecked

      There have been concerns raised by politicians and experts about the hoarding of Rs 2000 notes for illegal activities such as terror funding and drug trafficking. These concerns have sparked debates around phasing out the Rs 2000 denomination notes gradually. 

    • unchecked

      Banknotes go out of circulation due to soiling and mutilation. When notes become unfit for circulation, they are withdrawn and replaced. This natural process also affects the availability of specific denominations. 


    https://www.financialexpress.com/money/wherehave-the-rs-2000-notes-gone-finance-ministryclarifies-2911041/  


    1. Which of the following statement/s is/are correct regarding the ‘Forest-PLUS 2.0 Programme’?

    (1) This program is a five-year partnership between USAID and the Ministry of Environment, Forests and Climate Change

    (2) Forest-PLUS 2.0 Programme denotes ‘Forest for Wealth and Health’

    (3) Tetra Tech is building capacity across landscapes in three States in India

    (4) The three States which come under this program are; Andhra Pradesh, Karnataka, and Bihar

    (a) 2 and 4

    (b) 1 and 4

    (c) 1 and 3

    (d) 3 and 4

    Answer: C 

    Explanation: 

    Statement 1 is correct: The Forest-PLUS 2.0 program, launched by USAID and India's Ministry of Environment, Forest and Climate Change, is a significant initiative aimed at sustainable forest landscape management in India. The program, initiated in December 2018 and officially launched on September 25, 2019, focuses on developing tools and techniques to enhance ecosystem management and harness ecosystem services in forest landscape management. 

    Statement 2 is correct: The program focuses on developing tools and techniques to strengthen ecosystem-based management and the inclusion of ecosystem services in forest landscape management. It also aims to enhance the inclusive economic opportunities that emerge from improved landscape management. The targets of Forest-PLUS 2.0 are: 

    • 1,20,000 hectares of land under improved management. 

    •  New, inclusive economic activity worth $12 million. 

    • Measurable benefits accrued to 800,000 households. 

    The program includes pilot projects in three landscapes — Gaya in Bihar, Thiruvananthapuram in Kerala, and Medak in Telangana. These sites were chosen due to the contrast in their landscapes – Bihar is a forest deficit area, Telangana is a relatively drier area where there is ample scope for community livelihood enhancement, and Kerala is rich in biodiversity. 


    So, indeed, the Forest-PLUS 2.0 program signifies ‘forest for health and wealth’, as it aims to improve both the health of the environment and the wealth of the communities that depend on it. 


    Statement 3 is correct: The program is implemented by Tetra Tech ARD, a US-based consulting and engineering company, and its implementation partner, IORA Ecological Solutions, a New Delhi-based environmental advisory group. 


    Statement 4 is incorrect: Forest-PLUS 2.0 comprises pilot projects in three diverse 

    landscapes in India – Gaya in Bihar, Thiruvananthapuram in Kerala, and Medak in Telangana. These locations were chosen to address specific challenges and opportunities related to forest management. Bihar represents a forest deficit area, Telangana represents a relatively drier area with potential for community livelihood enhancement, and Kerala is rich in biodiversity. 


    Statements 1,2 and 3 are correct, and as per the given option the correct answer will be C) 1 and 3 

    https://www.downtoearth.org.in/news/forests/centre-us-agency-launch-forest-plus-2-0-66951


    1. Which one of the following statements is not correct in respect of ‘Euxine Sea’?

    (a) This Sea is also known as ‘Red Sea’.

    (b) It is located between Eastern Europe and Western Asia.

    (c) It serves as a barrier for NATO to counter Russia.

    (d) The earliest known name of the Sea is ‘the Sea of Zalpa’.

    Answer: A 

    Explanation: 

    The Euxine Sea, also known as the Black Sea, is not the same as the Red Sea. The Euxine Sea is a large inland sea located in southeastern Europe and southwestern Asia. It is bordered by Turkey, Bulgaria, Romania, Ukraine, Russia, Georgia, and Abkhazia. The Red Sea is a saltwater arm of the Indian Ocean located between Africa and Asia. It is bordered by Egypt, Sudan, Eritrea, Ethiopia, Djibouti, Yemen, Saudi Arabia, Jordan, and Israel. The Black Sea is also known as the Euxine Sea. It is located between Eastern Europe and Western Asia. It is bordered by six countries – Russia, Ukraine, Georgia, Turkey, Bulgaria and Romania. 

    https://en.wikipedia.org/wiki/Black_Sea#:~:text=Th 

    e%20earliest%20known%20name%20of,on%20th e%20Black%20Sea%20coast.%22


    1. Which of the following statement/s is/are correct in respect of ‘The First Ever Census of Water bodies in India-2023’ released by Ministry of Jal Shakti?

    (1) The Census enumerated a total 24,24,540 water bodies across the country

    (2) West Bengal has the highest number of Tanks

    (3) Tamil Nadu has the highest number of Lakes

    (4) Andhra Pradesh leads in water conservation schemes.

    (a) 1 and 2

    (b) 2 and 4

    (c) 1 and 3

    (d) 4 only


    Answer: C 

    Explanation: 

    Statement 1 is correct: As per the report of India's first water bodies census, a total of 24,24,540 water bodies have been enumerated in the country. This comprehensive census provides detailed information about ponds, tanks, lakes, reservoirs, and other water bodies across all states and Union Territories in India. 


    Statement 2 is incorrect: Andhra Pradesh leads in the number of tanks. Tanks are essential water storage structures, especially in regions where regular water supply is a challenge. They support agricultural activities and provide water for various other purposes. West Bengal has the highest number of ponds and reservoirs. Ponds and reservoirs play a vital role in storing water, especially in agricultural areas, aiding in irrigation and sustaining local ecosystems. 


    Statement 3 is correct: Tamil Nadu has the highest number of lakes. Lakes are significant natural water bodies, often serving as sources of drinking water, supporting biodiversity, and offering recreational and tourism opportunities. 


    Statement 4 is incorrect: Maharashtra is recognized as the leading state for water

    conservation schemes. This highlights the state's efforts and initiatives in implementing schemes aimed at conserving and managing water resources effectively. 


    Andhra Pradesh leads in the number of tanks. Tanks are essential water storage structures, especially in regions where regular water supply is a challenge. They support agricultural activities and provide water for various other purposes. 


    https://pib.gov.in/PressReleaseIframePage.aspx?PRID=1919482#:~:text=In%201st%20census%20of%20water,depicted%20from%20chart%20given%20below


    1. Which of the following statements are correct in respect of ‘Nomadic Elephant’?

    (1) It is an Indo-Mongolian joint military exercise

    (2) The Fourteenth Edition of this exercise took place in  the year 2019

    (3) This exercise is aimed to train Elephants in Defence activities

    (4) The First Edition of ‘Nomadic Elephant’ took place in Gwalior, Madhya Pradesh.

    (a) 1 and 3

    (b) 2 and 4

    (c) 2 and 3

    (d) 1 and 2


    Answer: D 


    Explanation: 

    Statement 1 is correct and Statement 3 is incorrect: The NOMADIC ELEPHANT is a joint military exercise between the Indian Army and the Mongolian Armed Forces. It is held annually, alternating between India and Mongolia. The exercise aims to enhance cooperation and interoperability between the two armies, and to focus on counter-terrorism and peacekeeping operations. The 15th edition of the exercise was held from July 17 to July 31, 2023, in Ulaanbaatar, Mongolia. 


    Statement 2 is correct: The 14th edition held at the Special Forces Training School, Bakloh, in October 2019 underscores the longstanding history of this joint military training event.


    Statement 4 is incorrect: The first exercise, Nomadic Elephant I, took place in 2004 in Mongolia. 


    https://www.livemint.com/news/world/indiamongolia-to-commence-joint-military-exercise-nomadicelephant-2023-11689489319004.html


    1. Which of the following statement/s is/are correct in respect of ‘Batagaika Crater’?

    (1) The Batagaika Crater is situated in Australia Far East

    (2) This region has become a focal point for studying the effects of climate change on permafrost.

    (3) The Batagaika Crater is affectionately referred as ‘The Sky-in’

    (4) The permafrost thaws release enormous quantities of Organic Carbon.

    (a) 2 and 3

    (b) 1 and 4

    (c) 1 and 3

    (d) 2 and 4


    Answer: D 

    Explanation: 

    Statements 1 and 3 are incorrect: The Batagaika crater, also known as the "megaslump" or the "gateway to the underworld," is a massive gash in the landscape in the Sakha Republic, Russia. It is the largest permafrost crater in the world, and it is growing rapidly. 

    Statements 2 and 4 are correct: Batagaika crater is a serious threat to both Russia and the planet as a whole. The melting permafrost is causing the ground to sink, which could damage infrastructure and displace people in northern and northeastern Russia. The release of organic carbon into the atmosphere is also a major concern, as it could further fuel global warming. 


    The Batagaika crater is a reminder of the urgent need to address climate change. If we do not take action to reduce greenhouse gas emissions, the impacts of climate change will become more severe and widespread. 


    https://www.wionews.com/photos/in-pics-gateway-to-underworld-worlds-biggest-permafrost-crater-thaws-due-to-climate-change-618542


    1. Which of the following statements are correct in respect of ‘Talisman Sabre Exercise – 2023’?

    (1) USA and France have initiated the ‘Talisman Sabre Exercise – 2023’

    (2) It is a joint military exercise of the two countries

    (3) This Exercise is held for every Four Years

    (4) India attended as an Observer to this military exercise

    (a) 2 and 4

    (b) 1 and 3

    (c) 3 and 4

    (d) 2 and 3

    Answer: A 

    Explanation: 

    Statement 1 is incorrect and Statement 2 is correct: Talisman Sabre is the largest bilateral military exercise between Australia and the United States, with multinational participation. It is designed to advance a free and open Indo-Pacific by strengthening relationships and interoperability among key allies and enhancing collective capabilities to respond to a wide array of potential security concerns. 


    Statement 3 is incorrect: The exercise is held every two years, alternating between Australia and the United States. 


    Statement 4 is correct: Military personnel from India, the Philippines, Singapore, and Thailand joined as observers at the exercise  

    https://en.wikipedia.org/wiki/Exercise_Talisman_Sabre

    1. Which of the following statements are true in relation to the G20 New Delhi Declaration?

    1. All 83 paragraphs of the 2023 G20 New Delhi Leaders’ Declaration were unanimously approved.

    2. It didn’t feature the conflict in Ukraine and its subsequent economic implications.

    3. The declaration stressed the urgency of mobilizing US$4 trillion per year for clean energy technologies by 2030.

    (a) 1 and 2 only

    (b) 2 and 3 only

    (c) 1 and 3 only

    (d) 1, 2 and 3


    Answer: C. 

    Explanation: 

    Statement 2 is incorrect: Prime Minister had said that India's presidency must be inclusive, it must be decisive, it must be ambitious and it must be action-oriented. The New Delhi Leader's declaration has 83 paras in all; all 83 paras have 100% consensus across all countries. It highlighted the human suffering and negative added impacts of the war in Ukraine with regard to global food and energy security, supply chains, macro-financial stability, inflation and growth, which has complicated the policy environment for countries, especially developing and least developed countries which are still recovering from the COVID-19 pandemic and the economic disruption which has derailed progress towards the SDGs. There were different views and assessments of the situation. Regarding climate change, the declaration stressed the urgency of mobilizing “US$5.8-5.9 trillion in the pre-2030 period for developing countries” and “US$4 trillion per year for clean energy technologies by 2030” to attain net-zero emissions by 2050.  


    https://www.mea.gov.in/media-briefings.htm?dtl/37093/Transcript_of_Press_brief


    https://www.india-briefing.com/news/key-outcomes-of-the-2023-g20-summit-held-in-india-29483.html/#:~:text=Regarding%20climate%20ch

    Economic and Social development-sustainable development, poverty, inclusion, Demographics, Social Sector initiatives, etc.



    1. MGNREGS is recognized as ‘core of the core’ scheme for achieving Sustainable Development Goals in India. MGNREGS was reported in the Voluntary National Review (VNR), 2017 by Government of India for its contribution to achieving which of the following SDGs directly?

    1. SDG1 – No Poverty

    2. SDG2 – Zero Hunger

    3. SDG5 – Gender Equality

    4. SDG10 – Reduced Inequalities

    (a) 1, 2, 3

    (b) 1, 3, 4

    (c) 2, 3, 4

    (d) 1, 2, 3, 4

    Answer: B 

    Explanation: 

    The SDGs and targets have been mapped to various nodal ministries and flagship programmes by the NITI Aayog. Ministry of Rural Development (MoRD) is the designated nodal agency for achieving SDG 1 – No Poverty, and MGNREGS has been recognised as ‘core of the core’ scheme for achieving SDG 1. MGNREGS was reported in the VNR 2017 by Government of India for its contribution to achieving SDG 1, while the scheme’s contributions to SDG 5 – Gender Equality, SDG 8 – Decent Work and Economic Growth, and SDG 10 – Reduced Inequalities were also acknowledged.  


    https://hlpf.un.org/countries/india/voluntary-national-review-2017



    1. Consider the following statements:

    1. Poverty line estimation in India has been based on the consumption expenditure and not on the income levels.

    2. It is difficult to assess incomes of self-employed people, daily wage laborers etc., and there may be large fluctuations in income due to seasonal factors

    Which of the following is correct in respect of the above statements?

    (a) Both 1 and 2 are correct and 1 is the correct explanation of 2

    (b) Both 1 and 1 are correct, and 2 is the correct explanation of 1

    (c) Both the statements are correct but none is an explanation of the other

    (d) Both the statements are correct but unrelated

    Answer: B 

    Explanation: 

    In India, poverty lines have historically been estimated based on consumption expenditure rather than income levels. The method used is known as the consumption-based approach. The official poverty line in India is determined by the Planning Commission (now NITI Aayog) and is based on the methodology recommended by various expert groups. 

    The poverty line is set by estimating the minimum level of consumption expenditure required to meet basic needs such as food, education, health, and other essential expenses. Assessing the incomes of self-employed individuals, daily wage laborers, and others in the informal sector can be challenging.


    https://prsindia.org/theprsblog/more-privatisation-on-the-cards


    1. Which of the following terms are the components of forex reserves?

    1. Foreign Currency Assets

    2. Gold

    3. Special Drawing Rights

    4. The Reserve in the International Monetary Fund 

    (a) 1, 2 and 3

    (b) 1, 2 and 4

    (c) 2, 3 and 4

    (d) 1, 2, 3 and 4

    Answer: D 

    Explanation: 

    Foreign Currency Assets (FCAs): These are holdings of foreign currencies like the US dollar, Euro, Pound Sterling, etc. FCAs form the largest component of forex reserves. Gold Reserves: This includes the gold held by the Reserve Bank of India (RBI) as part of its reserves. The value of gold in the reserves is subject to changes in international gold prices. 

    Special Drawing Rights (SDRs): SDRs are international reserve assets created by the International Monetary Fund (IMF) and allocated to its member countries. They represent a potential claim on freely usable currencies of IMF members. 

    Reserve Position in the IMF: This is the value of the reserve assets held by the RBI in the International Monetary Fund. Other Reserve Assets: This category includes any other reserve assets held by the RBI, such as foreign exchange assets with other central banks. 

    https://www.iasgyan.in/daily-current-affairs/forex-reserves-35


    1. Which of the following statements is NOT true in relation of PM-VISHWAKARMA?

    (a) It was launched by the President of India on 17th September, 2023.

    (b) It is a Central Sector Scheme

    (c) The new scheme intends to provide recognition and holistic support to the traditional artisans & craftsmen working with their hands and elementary tools

    (d) The scheme intends to improve the quality, scale and reach of artisans’ products and also to integrate them with MSME value chains


    Answer: A 

    Explanation: 

    Statement 1 is incorrect: The Prime Minister recently launched the Central Sector Scheme “PM Vishwakarma Scheme” to provide collateral-free loans and skills training to 18 categories of traditional artisans and craftspersons. The scheme has a total outlay of Rs 13,000 crore over five years, spanning until 2027-28. PM Vishwakarma, a Central Sector Scheme, was launched on 17th September, 2023 by the Prime Minister to provide end-to-end support to artisans and craftspeople who work with their hands and tools. 

    Statements 2 and 3 are correct: 

    • Objective: The PM Vishwakarma Scheme is designed to support traditional artisans and craftsmen by offering them financial assistance, skills training, and toolkits to enhance their craft and livelihoods. The Ministry of Micro, Small & Medium Enterprise is the nodal ministry of the PM Vishwakarma Yojana. 

    • Components of the Scheme:  Skills Training: Beneficiaries will attend a five-day skilling workshop.  

    • Toolkits: Artisans will receive a voucher of Rs 15,000 to purchase necessary tools for their craft.  

    • Interest-Free Loans: Artisans can avail themselves of an interest-free loan of Rs 1 lakh, repayable within 18 months. Upon repayment, they qualify for a second loan of Rs 2 lakh at an interest rate of 5 percent.  

    • Subsidy: Beneficiaries will receive a subsidy of up to 8 percent of the interest amount.  Credit Guarantee: The scheme offers guarantee-free loans.  

    https://pmvishwakarma.gov.in/

    1. With reference to GST, which of the following statements are true?

    1. GST is based on a system of concurrency of indirect taxes, where every transaction attracts central as well as state GST

    2. GST replaced a production-based taxation system by a consumption-based tax system

    3. It brings destination principle for cross-border trading for which the poorer and consuming states are benefiting at the expense of more affluent and industrialized ones

    (a) Only 1 and 2 are true

    (b) Only 1 and 3 are true

    (c) Only 2 and 3 are true

    (d) All the above are true


    Answer: D 

    Explanation: 

    • Statement 1 is correct: The Goods and Services Tax (GST) is based on the system of concurrency of indirect taxes. GST is a comprehensive indirect tax that subsumes a range of central and state taxes. It was introduced in India to create a unified and concurrent system of taxation, replacing various taxes like excise duty, service tax, valueadded tax (VAT), and others. 

    • Statement 2 is correct: The Goods and Services Tax (GST) is designed to shift the tax system from production-based to consumption-based. Under the GST system, taxes are levied at each stage of the supply chain, and they are ultimately borne by the end consumer. This transition from a production-based system to a consumption-based system aims to make taxation more equitable and reduce the cascading effect of taxes 

    • Statement 3 is correct: In the pre-GST era, manufacturing states often benefited more from taxes, and consuming states received less. The introduction of GST sought to balance this by ensuring that states where goods and services are consumed receive a fair share of tax revenue. This has generally been seen as a way to benefit less industrialised or economically weaker states, helping them with additional revenue for development and welfare programs. Meanwhile, industrialised states may see a decrease in tax revenue under this system  

    1. Which of the following statements is incorrect about Production Linked Incentive (PLI) scheme?

    (a) It is one of the several schemes as part of the ‘Make in India’ initiative.

    (b) Its aim is to incentivize manufacturers and promote domestic production.

    (c) This initiative contributes to enhancing self-reliance in the country.

    (d) This scheme is applicable only to pharmaceuticals, textiles and food processing sectors.

    Answer: D 

    Explanation: 

    • The 14 production-linked incentive (PLI) schemes are a key initiative of the central government to boost domestic manufacturing and exports in various sectors. These schemes aim to provide financial incentives to eligible companies based on their incremental production and sales. The Department for Promotion of Industry and Internal Trade (DPIIT) is the nodal agency for implementing these schemes and disbursing the incentives. 

    • Targeted Sectors: The 14 sectors are mobile manufacturing, manufacturing of medical devices, automobiles and auto components, pharmaceuticals, drugs, specialty steel, telecom & networking products, electronic products, white goods (ACs and LEDs), food products, textile products, solar PV modules, advanced chemistry cell (ACC) battery, and drones and drone components. Hence option D is incorrect. 


    1.  In discharging its role as a facilitator for rural prosperity, NABARD is entrusted with responsibilities of

    (a) Providing refinance to lending institutions in rural areas.

    (b) Bringing about or promoting development of commercial banks.

    (c) Evaluating, monitoring and inspecting all banks in rural areas.

    (d) Monitoring all developmental projects in rural areas.


    Answer: A 

    Explanation: 

    National Bank for Agriculture and Rural Development (NABARD) was established on 12 July 1982 by an Act of the Parliament. NABARD, as a Development Bank, is mandated for providing and regulating credit and other facilities for the promotion and development of agriculture, small scale industries, cottage and village industries, handicrafts and other rural crafts and other allied economic activities in rural areas with a view to promoting integrated rural development and securing prosperity of rural areas, and for matters connected therewith or incidental thereto. VISION: Development Bank of the Nation for Fostering Rural Prosperity. MISSION: Promote sustainable and equitable agriculture and rural development through participative financial and non-financial interventions, innovations, technology and institutional development for securing prosperity.

    https://www.nabard.org/about-departments.aspx?id=5&cid=466#:~:text=NABAR


    1.  India is one of the few countries experiencing falling child sex ratio. Which among the following statements could be the major reasons behind this phenomenon?

    1. High female infant mortality (relative to male infant mortality).

    2. Female foeticide.

    (a) Only 1

    (b) Only 2

    (c) Both 1 and 2

    (d) None of these


    Answer: B 

    Explanation: 

    • Sex-Selective Practices: One of the primary reasons is the practice of sexselective abortions, often driven by a preference for male children. The availability of ultrasound technology for prenatal sex determination has facilitated this practice despite being illegal. 

    • Socio-Cultural Factors: Deep-rooted socio-cultural norms and preferences for male heirs can contribute to discrimination against female children. In some communities, there is a historical preference for male offspring for reasons related to inheritance, dowry, and carrying on the family name. 

    • Son Preference: The preference for male children, often driven by beliefs about male lineage continuity, financial responsibilities related to dowry, and perceived social status, contributes to the imbalance. 

    • Dowry System: The dowry system, prevalent in many parts of India, places a financial burden on the families of girls during marriages. This can be a significant factor in families preferring male children to avoid future dowry expenses.  

    https://timesofindia.indiatimes.com/india/indias-female-imr-drops-to-same-level-as-males/articleshow/94629223.cms


    1. With reference to inflation targeting by the Reserve Bank of India, consider the following statements:

    1. In May 2016, the RBI Act, 1934 was amended to provide a statutory basis for the implementation of the flexible inflation targeting framework.

    2. Under Section 45ZA, the Central Government, in consultation with the RBI, determines the inflation target in terms of the Consumer Price Index (CPI), once in five years.

    3. On March 31, 2021, the Central Government retained the inflation target of 6 percent with the tolerance band of +2 percent for the next 5-year period -  April 1, 2021 to March 31, 2026.

    Which of the above statements are correct?

    (a) 1 and 2 only

    (b) 1 and 3 only

    (c) 2 and 3 only

    (d) 1, 2 and 3


    Answer: D 

    Explanation: 

    In May 2016, the Reserve Bank of India (RBI) Act, 1934 was amended to provide a Constitutional basis for the implementation of the flexible inflation targeting (FIT) framework. 

    Section 45-ZA of the RBI Act, 1934 requires that the Central Government shall, in consultation with the Reserve Bank of India (RBI), determine the inflation target in terms of a consumer price index (CPI), once in every five years. 

    On March 31, 2021, the government retained the inflation target and the tolerance band for the next 5-year period – April 1, 2021 to March 31, 2026.  


    https://indianexpress.com/article/business/economic-activity-higher-prices-lift-gst-mop-up-44-pc-7948213/


    1. Which of the following statements are not correct in respect of ‘Rajasthan Minimum Guaranteed Income Bill – 2023’

    (1) Under this Scheme every family is entitled to guaranteed employment for 125 days per year.

    (2) This Scheme provides guaranteed minimum monthly Pension fixed at Rs. 2000/- per month for specific categories

    (3) This scheme encompasses the flagship of ‘Jawahar Rozgar Yojana’ for urban areas.

    (4) To implement this programme Government has estimated additional expenditure of Rs. 2,500 Crore annually.

    (a) 3 and 4

    (b) 2 and 3

    (c) 1 and 3

    (d) 2 and 4


    Answer: B 

    Explanation: 

    The Rajasthan Minimum Guarantee Bill 2023 

    • Under the bill, all families in Rajasthan will be eligible to receive a minimum monthly income of Rs. 1,000. Families with unemployed members will be eligible to receive an additional Rs. 255 per day for up to 125 days per year. Statement 1 is correct. 

    • The bill also guarantees a minimum pension of Rs. 1,000 per month to the elderly, the disabled, and widows. This pension will be increased by 15% every year. Statement 2 is incorrect 

    • The bill adds 25 additional days of rural employment guarantee, making it a total of 125 days, over and above the 100 days provided by the MGNREGA. Statement 3 is incorrect 

    • The Rajasthan government has allocated an additional expenditure of Rs 2,500 crore for FY24 in the state budget for the minimum guaranteed scheme. Statement 4 is correct

    https://www.ndtvprofit.com/business/rajasthans-minimum-guaranteed-income-bill-showing-the-way-or-a-bad-precedent


    1. Which of the following statements are not correct?

    According to the Second Edition of National Multidimensional Poverty Index-2023 released by NITI Aayog – 

    (1) 13.5 Crore people escaped from multidimensional poverty

    (2) In rural areas there is rapid decline in poverty from 32.50% to 19.28%

    (3) Bihar registered the largest decline in the number of poor individuals.

    (4) This Report covered a comprehensive analysis of poverty across 1075 Administrative Districts

    (a) 2 and 4

    (b) 1 and 2

    (c) 3 and 4

    (d) 1 and 3

    Answer: C 

    Explanation: 

    Statements 3 and 4 are incorrect 

    National Multidimensional Poverty Index (MPI) report released by the Niti Aayog in 2023. 

    •  India experienced a substantial decline of 9.89 percentage points in the number of multidimensionally poor people, dropping from 24.85% in 2015-16 to 14.96% in 2019- 2021. This means about 13.5 crore (135 million) people moved out of multidimensional poverty during this period. 

    •  Rural areas witnessed a faster decline in poverty, decreasing from 32.59% to 19.28%. In urban areas, multidimensional poverty reduced from 8.65% to 5.27%. 

    •  States like Bihar, Uttar Pradesh, Madhya Pradesh, Odisha, and Rajasthan contributed significantly to the decrease in rural poverty. Uttar Pradesh saw the largest decline in the number of poor, with 3.43 crore (34.3 million) people escaping multidimensional poverty. 

    •  Delhi, Kerala, Goa, and Tamil Nadu, along with the Union Territories, had the least number of people facing multidimensional poverty. 

    •  The report used 12 parameters including health, education, and standard of living, such as nutrition, child and adolescent mortality, maternal health, years of schooling, school attendance, cooking fuel, sanitation, drinking water, electricity, housing, assets, and bank accounts.  

    https://www.thehindu.com/news/national/niti-aayog-report-claims-decrease-in-multidimensional-poverty/article67091078.ece


    1. Which State Government in India has taken significant step to introduce an educational imitative called ‘CM Rise Schools’?

    (a) Madhya Pradesh

    (b) Uttar Pradesh

    (c) Kerala 

    (d) Andhra Pradesh


    Answer: A 

    Explanation: 

    The Madhya Pradesh government has announced the establishment of 9,000 'CM Rise' schools across the state. The goal of the CM Rise schools is to provide high-quality education to all students, regardless of their background. The CM Rise schools will be equipped with stateof-the-art facilities, including smart classrooms, well-equipped laboratories, and libraries. The schools will also have highly qualified and trained teachers. 

    The CM Rise schools will focus on the holistic development of students, including their academic, social, and emotional well-being. The schools will also promote values such as patriotism, citizenship, and social responsibility. 


    https://www.indiatoday.in/education-today/news/story/madhya-pradesh-government-to-establish-9000-cm-rise-schools-2408419-2023-07-18


    1. Which of the following statements is correct in respect of the ‘Agreement on the use of Local Currencies for cross-border transactions’?

    (1) India and UAE have signed a memorandum in this Agreement.

    (2) This agreement enables a framework for trade settlements in their respective national currencies.

    (3) This Agreement was signed by the Prime Ministers of India and UAE

    (4) Peekay Intermark Ltd. And Reserve Bank of India were the First users of this system

    (a) 2 and 4

    (b) 3 and 4

    (c) 1 and 2

    (d) 1 and 3


    Answer: C 

    Explanation: 

    Statements 1 and 2 are correct 

    Local Currency Settlement System (LCSS) Agreement 

    •  India and the UAE signed a groundbreaking Memorandum of Understanding (MoU) to establish a Local Currency Settlement System (LCSS). This system allows trade settlement between the two nations in their respective national currencies – the Indian Rupee (INR) and the UAE Dirham (AED). This agreement covers all current account transactions and permitted capital account transactions. 

    •  The primary goal of LCSS is to promote the use of INR and AED for cross-border transactions, allowing exporters and importers to invoice and pay in their domestic currencies. This simplifies transactions and reduces dependency on third-party currencies. 

    •  MoU that was signed between the RBI Governor Shaktikanta Das and his counterpart at the Central Bank of UAE (CBUAE) Governor Khaled Mohamed Balama to promote the use of local currencies, the Indian Rupee (INR) and the UAE Dirham (AED), for cross-border transactions. 

    •  Peekay Intermark Ltd. and YES BANK have become the first entities to settle a gold transaction in Indian Rupees under the newly set up Local Currency Settlement (LCS) system between India and the UAE. 

    https://www.deccanherald.com/business/explained-india-uae-agreement-on-trade-settlement-in-local-currencies-1237454.html


    1. Which of the following statements is correct with respect to the ‘Namda Project’?

    (1) This project is a part of ‘Pradhan Mantri Kaushal Vikas Yojana’.

    (2) Under this programme 2200 candidates from Assam were trained

    (3) The aim of this project is to preserve the endangered crafts of wool weaving.

    (4) This project works under public-private partnership

    (a) 2, 3 and 4

    (b) 1, 3 and 4

    (c) 1, 2 and 3

    (d) 1, 2 and 4


    Answer: B 

    Explanation: 



    Namda Art Project in Jammu and Kashmir 

    • The Namda craft of Kashmir involves making rugs from sheep wool using the felting technique instead of traditional weaving. However, this craft was facing decline due to low availability of raw materials, lack of skilled manpower, and marketing challenges. Between 1998 and 2008, the export of Namda craft had almost diminished by 100%.

    • The Namda Art Project was launched as part of the Pradhan Mantri Kaushal Vikas Yojana (PMKVY) in 2021. 

    • The project aimed to preserve this endangered craft by training individuals in the art of Namda craft. The Ministry of Skill Development and Entrepreneurship (MSDE) designed a short-term training curriculum to teach the craft. The training program was implemented in 25 batches over three cycles, with each cycle taking approximately 14 - 16 months to complete. 

    • The project was implemented in collaboration with local industry partners, including Mir Handicrafts and Srinagar Carpet Training and Market Centre, showcasing a successful public-private partnership (PPP) model in the field of skill development. 

    • Nearly 2,200 candidates from six districts of Kashmir, namely Srinagar, Baramulla, Ganderbal, Bandipora, Budgam, and Anantnag, received training in the Namda craft.  

    https://pib.gov.in/PressReleaseIframePage.aspx?PRID=1939608


    History of Odisha and Odia Nationalism


    1. The Bhaumakaras started on era in the year 736 CE. What was the last known date mentioned in the Bhauma-kara records?

    (a) 187

    (b) 190

    (c) 204

    (d) 213

    Ans.: (d)

    The question was not understood properly.


    1. The Bhajas of Khinjali-mandala initially had their capital at Dhritipura, but later on they shifted their capital to

    (a) Vanjulvaka 

    (b) Vinitapura 

    (c) Yayatinagara 

    (d) Vyaghrakota 


    Answer: C 

    Explanation: 

    The Sonepur Copper Plate of Satrubhanja Dev mentions the name Khinjali Mandala for the first time. This document is believed to connect Silabhanja Dev to the founding of the Bhanja Kula in Khinjali Mandala. The capital of this region was Dhirtipura, identified with modern-day Baudh town. 


    Raja Satrubhanja Dev II was defeated and killed by the Somavansi ruler of South Kosala, Raja Janmeyjaya I. As a result, the Bhanjas were driven out from the Baudh region, which was renamed Odra Desa. Raja Yayati I, the successor of Janmejaya I, established his capital in Odradesa at Yayatinagara, identified with modern Jagati in Baudh District. 


    https://oroyalarchives.com/baudh/


    1. With reference to Mahasivagupta Balarjuna consider the following:

    1. He was the Panduvamsi king of South Kosala with is capital located at Sirpur.

    2. He extended his dominion by capturing Suvarnapura, modern Sonepur.

    3. He ruled for more than 57 years.

    How many statements given above are correct?

    (a) Only one

    (b) Only two

    (c) All three

    (d) None 


    Answer: C 

    Explanation: 

    Maha-Shivagupta Balarjuna 

    ● Maha-Shivagupta Balarjuna, also known as Shivagupta, was the greatest and most powerful king of the Panduvamshi dynasty. He was referred to as Maha-Shivagupta in his own copper-plate inscriptions and as Shivagupta or Balarjuna in other inscriptions. Balarjuna was likely his personal name, while Mahashivagupta was his coronation name. 

    ● Maha-Shivagupta's reign marked a golden age for Sirpur, the capital of the Panduvamshis in Dakshina Kosala. Under his rule, Sirpur experienced significant cultural and political growth. Different religions received royal patronage, leading to the construction of various temples and monasteries. Notably, he built the important Buddhist monastery at Sirpur and the famous temple of Surang Tila. His reign was peaceful and prosperous, possibly lasting for at least 57 years, as indicated by an inscription from his 57th regnal year.


    https://en.wikipedia.org/wiki/Maha-Shivagupta_Balarjuna


    1. With reference to the creation of a separate province of Orissa consider the following statements.

    1. The British Parliament’s Constitution of Orissa order, 1936 got the approval of the king on 3 March 1936 and Orissa became a new province on 1 April 1936.

    2. Sir John Austin Hubback was appointed as the first Governor of Orissa.

    3. Sir Courtney Terel, the Chief Justice of Bihar and Orissa High Court, administered the oath of office to the new Governor.

    4. The inaugural function was held in Ravenshaw College Hall at Cuttack.

    How many statements give above is/are correct?

    (a) Only one

    (b) Only two

    (c) Only three

    (d) All four


    Answer: D 

    Explanation: 

    Statement 1 is correct: The Constitution of Orissa Order, 1936, was approved by the British King on March 3, 1936. The Order was passed under the Government of India Act, 1935, which gave the British Crown the power to create new provinces in India. The Constitution of Orissa Order, 1936, created the new province of Orissa, which came into existence on April 1, 1936. The Order also established the legislative and executive councils for the new province. 

    Statement 2 is correct: On 1 April 1936, Bihar and Orissa were split into separate provinces. The new province of Orissa came into existence on a linguistic basis during the British rule in India, with Sir John Austen Hubback as the first governor. 

    Statement 3 is correct and Statement 4 is correct: 

    On the appointed day in a solemn ceremony held at the Ravenshaw College Hall, Cuttack, Sir John Austin Hubback was administered the oath of office as the first Governor of Orissa. 

    The oath was administered by the Chief Justice of the Bihar and Orissa High Court, Sir Courtney Terrell. It was a historic moment for the people of Orissa, as it was the first time that the state had its own governor.  


    https://apps.odishatourism.gov.in/blog-detail/story-of-creation-of-a-separate-province-or-modern-state-of-odisha#:~:text=His%20majesty%20issued%20the


    1. Mahatma Gandhi visited Odisha in

    1. March 1921

    2. August 9125

    3. December 1927

    4. September 1930

    Which of the above is not correct?

    (a) Only 1

    (b) Only 2

    (c) Only 3

    (d) Only 4


    Answer: D 

    Explanation: 

    Statement 4 is incorrect 

    Gandhi's visits to Odisha 

    These visits provide valuable insights into the significant role Odisha played in the Indian independence movement. Here's a summary of Gandhi's visits to Odisha during the mentioned periods: 

    • First Visit: March 23-29, 1921  Gandhi arrived in Cuttack and conducted meetings to propagate the ideas of the Noncooperation Movement.  He addressed women at Binod Bihari and a large crowd on the riverbed of Kathajodi.  Visited Bhadrak and addressed a meeting at Gandhi Padia.  Addressed a public meeting in Puri on freedom struggle.  Visited Brahmapur and addressed a gathering at the Brahmapur Stadium. 

    • Second Visit: August 19-20, 1925  Gandhi visited Utkal Tannery in Cuttack and other places before leaving for Kolkata. 

    • Third Visit: December 4-21, 1927  Promoted Khadi and visited Khadi production centers, including Brahmapur.  Met flood-affected people in Bhadrak district.  Attended a public meeting in Cuttack. 

    • Fourth Visit: December 22, 1928  Gandhi addressed a meeting in Jharsuguda and Sambalpur about various issues faced by the people. 

    • Fifth Visit: May 5-May 16, 1934  Conducted a Padyatra for the welfare of Dalits, addressing people in Jharsuguda, Sambalpur, Angul, Puri, and Cuttack. 

    • Sixth Visit: May 21-June 7, 1934  Resumed Harijan Padyatra, covering rural areas of Cuttack, Kendrapada, Bari, Kabirpur in Jajpur, and Bhadrak districts. 

    • Seventh Visit: March 25-31, 1938  Attended a week-long conference of Gandhi Seva Sangh and an agro and village enterprise exhibition at Beraboi in Puri district. Sardar Vallabhbhai Patel and Rajendra Prasad also attended. 

    • Eighth and Last Visit: January 20, 1946  Met supporters at Baleswar, Bhadrak, Cuttack, and Brahmapur while traveling towards Chennai by train. 


    These visits reflect the deep connection between Mahatma Gandhi and the people of Odisha during the struggle for India's independence. Gandhi's presence and interactions inspired thousands and played a vital role in the freedom movement in the region.  


    https://timesofindia.indiatimes.com/city/bhubaneswar/mahatma-gandhi-visited-odisha-eight-times/articleshow/71408753.cms


    1. Kapilendradeva subdued many feudatory chiefs who rebelled against him. Which of the following did not belong to this category?

    (a) The Gangas of Khemundi

    (b) The Chauhans of Patna

    (c) The Matsyas of Oddadi

    (d) The Silavamsis of Nandapura


    Answer: B 

    Explanation: 

    With his coronation, Kapilendra Deva established the Suryavamsa Gajapati dynasty/Routray Gajapati dynasty which played a pivotal role in medieval history of India. Soon he sorted out the various serious problems of KalingaIn the South, the Reddy rulers of Rajahmumdry were extending their frontiers further north. From the North, the kingdom was threatened by the Sultan of Bengal. The distant Malwa also tried to take advantage of the revolution of local chiefs and feudatories who refused to acknowledge the suzerainty of Kapilendra Deva. 

    At that critical juncture, Kapilendra Deva Routray took immediate steps and subdued the rebel chiefs like the Matsyas of Oddadi, the Salivamsi chiefs of Nandapura, the Vishnuvardhana Chakravartins of Panchadharala and the Gangas of Khimindi. His Lingaraj temple inscription thus reveals: “all kings of my kingdom of Odisha should work for the good of the paramount sovereign and should keep to virtuous ways and not remain in bad ways. If they act badly towards their sovereign, they will be expelled from the kingdom and all their property would be confiscated.” This order definitely directed the defiant chiefs to acknowledge the suzerainty of Kapilendra Deva.  


    https://tfipost.com/2021/06/kapilendra-deva-routray-the-legendary-odia-king-and-founder-of-suryavamsa-routray-gajapati-dynasty/#:~:text=Military%20campaigns%20of%2



    1. Which of the following pair is not correctly matched?

    (a) Bichitrananda Das : Kujibara Patrika

    (b) Utkal Dipika : Gauri Shankar Ray

    (c) Sambalpur Hitaisini : Nilamani Vidyaratna

    (d) Dainika Asha : Sashi Bhusana Rath


    Answer: A 

    Explanation: 

    August 4, 1866, the day of glory for odia journalism. ‘Utkal Deepika’ launched itself as Odisha’s first Odia newspaper. Gaurishankar Roy was its editor. It was first published as a weekly and later as a daily. The newspaper was published by the Cuttack Printing Company established in Cuttack by the efforts of Gaurishankar and his close associate Bichitrananda Das. Utkala Deepika played an important role in giving new directions in the social and political spheres of that time and presenting strong demands to the government for the self-defense of the Odia people. The newspaper made efforts for social reform by raising its voice against abhorrent practices like child marriage. Utkal Deepika’s success created a trend in newspaper publishing in Odisha. Therefore, every year August 4th is celebrated as Odisha Journalism Day. ‘Magazine’ has played an important role in Odia journalism. Its journey began with a manuscript magazine in 1769. Sadhusundar Das of Kujibar Math near Cuttack Choudwar started a handwritten journal in 1769 called ‘Kujibara Patrika’. At that time, the journey was started with the aim of informing the common people about the current situation and about the odia language. It is true that with time it is now reaching digital, but Kujibara patrika continue to carry its golden history. Gyanaruna was published in 1849 as the first print magazine after Kujibara patra. Later magazines were published under the name Prabodha Chandrika in 1856 and Arunodaya in 1861. But it could not last much longer. Within 3 years, the publication of the said magazines was stopped due to various reasons. Later, in 1873, a monthly magazine called Utkal Darpan was published from Balasore. It is the first literary magazine in Odia language. Later in 1878 Utkal Madhup was published, in 1885 Pradeep magazine was published for a short period of time. The Utkal Sahitya magazine published in 1897 ushered in a new era in Odia literature. Subsequently, several magazines were published. Mukur in 1906, Satyabadi in 1915, Sahakara in 1920, Navabharat in 1934 etc. Magazines were very helpful in popularizing the Odia language and literature


    https://odishabhaskar.in/odisha/odia-journalismday-669/  


    1. Where do we come across the madhavapura mahavihara?

    (a) Ratnagiri 

    (b) Udayagiri 

    (c) Lalitagiri 

    (d) Pushpagiri 


    Answer: B 

    Explanation: 

    Udayagiri is the largest Buddhist complex in the Indian state of Odisha. It is composed of major stupas and monasteries (viharas). Together with the nearby complexes of Lalitgiri and Ratnagiri, it is part of the "Diamond Triangle" of the "RatnagiriUdayagiri-Lalitgiri" complex. 

    It used to be thought that one or all of these were the Pushpagiri Vihara known from ancient records,[3] but this has now been convincingly located at a different site. Per epigraphical artifacts found at the site, Udayagiri's historical name was "Madhavapura Mahavihara." This Buddhist complex, preceded by the Ratnagiri and Lalitgiri sites, with their monasteries, is believed to have been active between the 7th and the 12th centuries. 

    https://en.wikipedia.org/wiki/Udayagiri,_Odisha



    1. What is not true about the Prajamandala movement in Odisha?

    (a) It was a movement against the obnoxious taxes and ill treatment to the subject population by the feudatory chiefs of the Gadjat states.

    (b) Sarangadhara Das, Harekrushna Mahatab, Pabitra Mohan Pradhan and Nabakrushna Choudhury were the important leaders of the movement.

    (c) It started with the murder of Major Bezelgate, the Political Agent in Ranapur.

    (d) A State Enquiry Committee was constituted by the Government under the chairmanship of Harekrushna Mahatab in 1937 to report on the condition of people in Gadjat states.

    Answer: C 

    Explanation: 

    When the province of Orissa was created by dividing the existing Bihar and Orissa Province in 1936, it became the first administrative region of British India to be defined on a linguistic basis. Although this satisfied some long-standing desires, it made little practical difference to many of the populace because the new province included 26 areas under independent feudal control, called garjats, where the peasantry in particular suffered the effects of misrule and autocratic powers. The remainder of the new province was controlled by the British Raj authorities. Although the people of Orissa had little involvement in the Gandhi-inspired Salt March protests of 1930, that campaign did lead to the First All Orissa States People's Conference being held at Cuttack in the following year, with Bhubanananda Das and Sena Adhaksya Senapati Nakula Samanta Sinhar of HindolPrincelyState acting as president. It had the aim of persuading the ruling garjat chiefs to agree to more representative government but achieved nothing more than to increase their repression. A second Conference held in 1937 had more effect. Its aim was to achieve more equitable government and it led to the establishment of Praja Mandals (People's Associations) in most of the garjat states, demanding changes to the feudal system. Their demands included an end to forced labour and illegal taxes, as well as rights relating to freedom of association, publication of newspapers and property. The first sign of trouble involving a Praja Mandal arose in the princely state of Nilgiri, where peasant demands for fairer laws resulted in the ruler adopting repressive, even brutal, measures in response. Whilst a temporary truce was eventually achieved, problems then arose also in the states of Talcher and Dhenkanal. While the ruler of the latter state took such strong measures that several people were killed, the ruler of Talcher faced an unusual protest that attracted national attention: many of his peasantry moved out of the state to settle in temporary camps in the British-governed area of Orissa.Another unusual protest occurred in Nayagarh State when its ruler attempted to introduce a monopoly on the supply of betel: the population reacted by almost entirely stopping their consumption, which led to the arrest of some of its leaders. Discontent was voiced in other states, such as Athgarh, Baramba, Narsinghpur, Nayagarh, Ranpur and Tigiria, sometimes resulting in violence and death. Following an organised "All Orissa Garjat Day" on 29 October 1938, when people attempted to voice their concerns in unity, the troubles continued into 1939. In early January 1939, a British Political Agent, Major Bazelgette, was killed while trying to defuse tensions in Ranpur. The news of this dampened protests elsewhere, and notably so in Nayagarh, where Bazelgette had been summoned a few days earlier to assist the beleaguered raja in calming the Praja Mandal. Whilst the rajas might resort to violence, the Indian independence movement, as exemplified by Gandhi's Congress Party, believed in non-violent protest and civil disobedience; Congress leaders such as Gandhi, Jawaharlal Nehru and Subash Chandra Bose welcomed the political awakening of the people in Orissa but simultaneously condemned the killing. The British responded to the death by sending in troops, executing the ringleaders, and deposing the raja in favour of direct control. 


    A 1939 report by the Orissa States Enquiry Committee, which had been formed in 1937 to investigate the garjat states, came to nothing because of the outbreak of World War II, during the period of which the British would not countenance any changes or agitation. The Praja Mandal movement came to the fore again when the war ended, spurred by the possibility of wideranging change should the British grant India its independence. The first cracks appeared when the ruler of Nilgiri attempted to put a wedge between the tribal and non-tribal communities of his state by fomenting attacks from one on the other. This decision, which he hoped would undermine the Praja Mandal, created a major problem in law and order that made it necessary for the Government of Orissa to intervene on instructions from the central government. The central government were particularly concerned to contain the problem, which had the potential to spread into neighbouring states, and which in part had also been engendered by the unwillingness of the newly formed Eastern States Union, comprising numerous princely states, to submit to Praja Mandal demands for democratic representation in its governance. Thus, Nilgiri came under the aegis of the provincial government from November 1947 and within weeks all but two of the other garjat states - Saraikela and Kharsawan - had agreed to merge with the main Orissa province. Prominent figures in the Orissa Praja Mandal movement included Sarangadhar Das, Baisnab Charan Pattnaik, Pabitramohan Pradhan, Nabakrushna Choudhury and Kailas Chandra Mohanty.


    https://historyofodisha.in/the-prajamandal-movement-in-odisha/


    1. Subei, Kachela and Jamuda, the three sites in the undivided Koraput district of Odisha, are famous for?

    (a) Jaina relics

    (b) Buddhist relics

    (c) Brahmanical relics

    (d) Neolithic tools


    Answer: A 

    Explanation: 

    Subai Jain temple complex is a group of five Jain temples built in 4th century. Subai was an important Jain center and the Jain temple were constructed Jain gemstone traders who came to Koraput region for trading. the temples are dedicated to Mahavira, Parshvanatha, Rishabhanatha and other Tirthankaras. Kechela: A village situated in the sourthen bank of Kolab at a distance of 10 Kms away from Koraput. There is a copper plate starting the fact that the village had been granted to one Narasingha Mishra. On the occasion of the Solar eclipse 24 September 1620 by Maharaja Krishna Dev. There is a Jain temple about 30 ft. high contains 05 images of thirthankars namely Resavanath, Mahavir Jain, Ambika Devi, Jakhya & Jakhyani. 


    History of India and Indian national movements.


    1. Consider the following statements regarding the Rig Veda.

    1. It mentions the Aryans as a cultural community.

    2. It has many commonalities with Avesta.

    3. It mentions Saraswati as naditarna.

    4. It indicates that the dasyus kept cattle for dairy products.

    Which of the following are correct?

    (a) 1, 2 and 3

    (b) 2, 3 and 4

    (c) 1, 3 and 4

    (d) All are correct

    Answer: A 

    Explanation: 

    In any case, scholars point out that, even in ancient times, the Aryan identity as asserted in the Rig Veda was cultural, religious, and linguistic, not racial. 

    Theories about the homeland of the Aryans have been in news of late because of genetic studies. The theory that ascribes an indigenous origin to the Aryans can be shown to be untenable on very simple considerations based on a comparative study of the Rig Veda (RV) and the related Zoroastrian sacred text Avesta (AV). The Rig Vedic and Avestan languages are essentially the same, with very minor differences in grammar. They share a common vocabulary in the fields of mythology, ritual, culture, and religious practices. There are some phonetic differences but the changes take place according to well-defined rules (Sanskrit s into h, h into z). Ahura in AV (as in Ahura Mazda) is cognate with asura in RV with the same meaning, lord (asura as a demon is a later development.) Yama son of Vivasvan is known to AV. Nabhanedishta is a son of Manu in RV; it becomes a common noun in AV meaning “nearest in relation”. Saraswati (Naditarna) is mentioned as the holiest river in Rig Veda, named after Harakwati (Helmand River) in south Afghanistan. In Rig Veda, the popular Indological Approach was to treat ‘dasas’ and ‘dasyus’ as ‘non-Aryans’ natives of India in opposition to ‘Aryans’. Dasa: 54 hymns and 63 verses and Dasyus: 65 hymns, 80 verses. The accounts of conflict between the Aryans and the Dasas and Dasyus are acknowledged in Rig Veda. It describes that Dasas and Dasyus were people who do not perform sacrifices or obey the commandments of God. Their speech is described as ‘Mishra’. Dasas and Dasyus were early Indo-Aryan immigrants who arrived before the Vedic Aryans into the subcontinent.


    1. Consider the following statements regarding the Parthians.

    1. They had migrated from Iran.

    2. They set up a kingdom in East India.

    3. Gondophernes was a Parthian king.

    4. They are mentioned together with the Shakas in the Sanskrit texts.

    Which of the following are correct?

    (a) 1, 2 and 3

    (b) 1, 3 and 4

    (c) 2, 3 and 4

    (d) Only 3 and 4


    Answer: B 

    Explanation: 

    The Indo-Parthian Kingdom was a Parthian kingdom founded by Gondophares, and active from 19 CE to c. 226 CE. At their zenith, they ruled an area covering parts of eastern Iran, various parts of Afghanistan and the northwest regions of the Indian subcontinent (most of modern Pakistan and parts of northwestern India). The rulers may have been members of the House of Suren, and the kingdom has even been called the "Suren Kingdom" by some authors. The kingdom was founded in 19/20 when the governor of Drangiana (Sakastan) Gondophares declared independence from the Parthian Empire. He would later make expeditions to the east, conquering territory from the Indo-Scythians and Indo-Greeks, thus transforming his kingdom into an empire. The domains of the Indo-Parthians were greatly reduced following the invasions of the Kushans in the second half of the 1st. century. They managed to retain control of Sakastan, until its conquest by the Sasanian Empire in c. 224/5. In Baluchistan, the Paratarajas, a local IndoParthian dynasty, fell into the orbit of the Sasanian Empire circa 262 CE. The Indo-Parthians are noted for the construction of the Buddhist monastery Takht-i-Bahi (UNESCO World Heritage Site) in Mardan, Pakistan. Shakas' domination in northwest India was followed by that of the Parthians. In many ancient Sanskrit texts, they are mentioned together as the ShakaPahlava.  


    https://en.wikipedia.org/wiki/Parthian_Empire



    1. Consider the following statements about Sangam literature.

    1. The narrative texts are called Melkanakku.

    2. The didactic texts are called Kilkanakku.

    3. Thirukkural is a major Sangam text.

    4. Many Sangam texts were the works of Brahmana scholars.

    Which of the following are correct?

    (a) 1, 2 and 3

    (b) 1, 2 and 4

    (c) 2, 3 and 4

    (d) Only 1 and 4

    Answer: A 

    Explanation: 

    The narrative texts of the Sangam literature are called Melkannakku. These are 18 major works consisting of eight anthologies and ten idylls. The didactic works are called Kilkanakku or Eighteen Minor. The Tirukkuṟaḷ, or shortly the Kural, is a classic Tamil language text consisting of 1,330 short couplets, or kurals, of seven words each. The text is divided into three books with aphoristic teachings on virtue, wealth and love, respectively. The Sangam literature was composed by 473 poets, some 102 anonymous. According to Nilakanta Sastri, the poets came from diverse backgrounds: some were from a royal family, some merchants, some farmers. At least 27 of the poets were women. These poets emerged, states Nilakanta Sastri, in a milieu where the Tamil society had already interacted and inseparably amalgamated with north Indians (Indo-Aryan) and both sides had shared mythology, values and literary conventions. 


    1. Consider the following pairs

    1. Amoghavarsha - A king and a Kannada poet.

    2. Pampa - Kannada Poet under Jain influence.

    3. Nanniah - Began writing the Mahabharata in Telugu.

    4. Ponna - Jain Poet, who wrote on Ramayana themes.

    Which of the pairs are correctly matched?

    (a) 1, 2 and 3

    (b) 2, 3 and 4

    (c) 2 and 4

    (d) All are correct



    Answer: D

    Explanation: 

    Amoghavarsha I (also known as Amoghavarsha Nrupathunga I) (r.814–878 CE) was the greatest emperor of the Rashtrakuta dynasty, and one of the most notable rulers of Ancient India. His reign of 64 years is one of the longest precisely dated monarchical reigns on record. Many Kannada and Sanskrit scholars prospered during his rule, including the great Indian mathematician Mahaviracharya who wrote Ganita-sarasamgraha, Jinasena, Virasena, Shakatayan and Sri Vijaya (a Kannada language theorist). Amoghavarsha I was an accomplished poet and scholar. He wrote (or co-authored) the Kavirajamarga, the earliest extant literary work in Kannada, and Prashnottara Ratnamalika, a religious work in Sanskrit. During his rule he held titles such as Nrupathunga, Atishadhavala, Veeranarayana, Rattamarthanda and Srivallabha. He moved the Rashtrakuta regal capital from Mayurkhandi in the Bidar district to Manyakheta in the Kalaburagi district in the modern Karnataka state. He is said to have built the regal city to "match that of Lord Indra". The capital city was planned to include elaborately designed buildings for the royalty using the finest of workmanship. The Arab traveler Sulaiman described Amoghavarsha as one of the "four great kings of the world." For his religious temperament, his interest in the fine arts and literature and his peace-loving nature, historian Panchamukhi has compared him to the emperor Ashoka and given him the honorific "Ashoka of the South".[5] Amoghavarsha seems to have entertained the highest admiration for the language, literature and culture of the Kannada people as testified to in the text Kavirajamarga. Ponna was a noted Kannada poet in the court of Rashtrakuta Dynasty king Krishna III. The emperor honoured Ponna with the title "emperor among poets" for his domination of the Kannada literary circles of the time, and the title "imperial poet of two languages" for his command over Sanskrit as well.  


    1. Consider the following statements about Buddhism.

    1. It was patronized by the Pala kings.

    2. Buddha was worshipped as a God in Mahayana Buddhism.

    3. Hinayana considers Buddha as an ordinary being who attained Nirvana.

    4. Vajrayana focused on Hindu Practices.

    Which of the following are correct?

    (a) 2, 3 and 4

    (b) 1, 2 and 3

    (c) Only 3 and 4

    (d) All are correct


    Answer: D 

    Explanation: 

    The Palas were patrons of Mahayana Buddhism. Mahayana: It is one of the two main schools of Buddhism. The term Mahayana is a Sanskrit word which literally means "Great Vehicle". It believes in the heavenliness of Buddha and Idol worship of Buddha and Bodhisattvas embodying Buddha Nature. The main difference between Hinayana Buddhism and Mahayana Buddhism is that Mahayana sects of Buddhists believe that Buddha is a divine being and he guides the path to 'Nirvana'. In contrast, Hinayana sects consider Buddha as a Human Being. Vajrayana, (Sanskrit: “Thunderbolt Vehicle” or “Diamond Vehicle”) form of Tantric Buddhism that developed in India and neighbouring countries, notably Tibet. Vajrayana, in the history of Buddhism, marks the transition from Mahayana speculative thought to the enactment of Buddhist ideas in individual life. It includes practices that make use of mantras, dharanis, mudras, mandalas and the visualization of deities and Buddhas.  


    1. Consider the following statements regarding Alauddin Khalji.

    1. He set up three distinct markets one for food grains, one for cloth and the third for horses.

    2. A Shahna was posted in the market.

    3. The Peasants had to pay land revenue in kind.

    4. Slaves were transacted in market during his time.

    Which of the following are correct?

    (a) 1, 2 and 3

    (b) 2, 3 and 4

    (c) 1, 2 and 4

    (d) Only 1 and 4


    Answer: C 

    Explanation: 

    After fixing the price of commodities, Alauddin went for division of market. He divided the market broadly into three categories. i) The Central grain market or Mandi with subsidiary control shops in every mohallas of the city. ii) The Ser-i-adl, an exclusive market for clothes and luxury items. iii) Market of slaves, horses and cattle. Alauddin Khilji appointed separate officials known as Sahaan-e-mandi to oversee markets and to 

    punish those who defied his regulations. Land revenue had to be paid in Cash or Kind: 50%.  


    1. Consider the following statements regarding the Subsidiary Alliance in British India.

    1. The Indian ruler signing the alliance would agree to the posting of a British Resident.

    2. The Indian ruler could employ any European in his state.

    3. The Indian ruler would cede a part of his territory for payment of subsidy.

    4. Expenses of the British army in the state to be shared with Company.

    Which of the following are correct?

    (a) 1, 2 and 3

    (b) 2, 3 and 4

    (c) 1, 2 and 4

    (d) Only 1 and 3


    Answer: D Explanation: A subsidiary alliance, in South Asian history, was a tributary alliance between an Indian state and a European East India Company. Under this system, an Indian ruler who formed a treaty (agreement) with the company in question would be provided with protection against any external attacks. In return, the ruler was required to:  


    • keep the company's army at the capital of their state,  

    • give either money or territory to the company for the maintenance of the troops,

    • expel all other Europeans from their state, based on criteria of whether they were employed in the army or in the civil service,  

    • keep a European official called 'resident' at the capital of their state who would oversee all negotiations and communications with other states, meaning that the ruler was to have no direct correspondence or relations with other states, without the resident's approval.

    The ruler was also forbidden from maintaining a standing army. Agents from the East India Company were hired to live in the places, they later started interfering in the internal affairs of the British administration, such as the next ruler or nawab.  


    1. Consider the following regarding the Revolt of 1857.

    1. Indore troops had joined the Revolt against the wish of their king.

    2. Mangal Pandey was executed at Meerut.

    3. Tantia Tope was active both at Kanpur and Jhanshi.

    4. Sindhia sought refuge from the British at Agra.

    Which of the following are correct?

    (a) 1, 3 and 4

    (b) 2, 3 and 4

    (c) Only 3 and 4

    (d) Only 1 and 4


    Answer: A 

    Explanation: 

    Maharajadhiraj Raj Rajeshwar Sawai Shri Sir Tukoji Rao II Holkar XI Bahadur. In the Indian Rebellion of 1857, Indore State stayed loyal to the British side. The flames of the first war of Indian independence spread rapidly in many parts of Central India as the sipahis as well as local rajas, zamindars, and their retainers rose up and attacked British military stations and cantonments. Indore too joined the movement, spearheaded by Saadat Khan, an officer in the cavalry unit of Maharaja Tukoji Rao Holkar II, the ruler of Indore. With other revolutionaries, Saadat Khan, strategically planned and executed an early morning surprise attack against the British officials in their Indore Residency. The British officials failed to resist the revolutionaries and it resulted in the killing of 39 people inside, while others fled along with Colonel Durand and Colonel Travers, abandoning the Residency. However, in subsequent battles, Saadat Khan was defeated by the overwhelming might of British arms, but managed to escape. He managed to evade capture and continued to wage further struggle till September 1874, when he was finally captured and executed. Mangal Pandey's execution took place on 8 April 1857, before all of the Indian and British units stationed in Barrackpore. After the rebellion in Cawnpore (Kanpur) broke out on 5 June 1857, Nana Saheb became the leader of the rebels. When the British forces at Cawnpore surrendered on 25 June 1857, Nana was declared Peshwa in late June. After a defeat, Nana's troops had to withdraw to Bithur, after which Havelock crossed the Ganges and retreated to Awadh. Tantia Tope began to act in Nana Saheb's name from Bithur. Tantia Tope was one of the leaders of the massacre of Cawnpore, which occurred on 27 June 1857. Afterwards, Tope held a good defensive position until he was driven out by the British force on 16 July 1857. Afterward, he defeated General Cyrill in the Second Battle of Cawnpore, which started on 19 November 1857 and continued for seventeen days. Tope and his army were defeated when the British counterattacked under Sir Colin Campbell. Tope and other rebels fled the scene and had to take shelter with the Rani of Jhansi, while aiding her as well. The Scindias were backing the Britishers. As a written proof, Rahamani shows page No. 152 from Bipin Chandra’s book “The History of Modern India” where Bipin Chandra writes, “She captured Gwalior with the help of Tantia Tope and her trusted Afghan guards. Maharaja Scindia, loyal to the Britishers, made an attempt to fight the Rani, but most of his troops deserted him. Scindia sought refuge with English at Agra.”


    1. Consider the statements regarding the nationalist Romesh Chunder Dutt.

    1. He translated Ramayana and Mahabharata.

    2. He was president of the Indian National Congress.

    3. He qualified the ICS.

    4. He was president of the Bongiya Sahitya Parishad.

    Which of the following statements are correct?

    (a) 1, 2 and 3

    (b) 2, 3 and 4

    (c) Only 2 and 3

    (d) All are correct


    Answer: D 

    Explanation: 

    Romesh Chunder Dutt was an Indian civil servant, economic historian, translator of Ramayana and Mahabharata. He was one of the prominent proponents of Indian economic nationalism. He entered the Indian Civil Service as an assistant magistrate of Alipur in 1871. A famine in Meherpur district of Nadia in 1874 and another in Dakhin Shahbazpur (Bhola District) in 1876, followed by a disastrous cyclone, required emergency relief and economic recovery operations, which Dutt managed successfully. He served as administrator for Backerganj, Mymensingh, Burdwan, Donapur, and Midnapore. He became Burdwan's District Officer in 1893, Commissioner (offtg.) of Burdwan Division in 1894, and Divisional Commissioner (offtg.) for Orissa in 1895. Dutt was the first Indian to attain the rank of divisional commissioner. Dutt retired from the ICS in 1897. He was the president of Indian National Congress in 1899. He was also a member of the Bengal Legislative Council. He served as the first president of Bangiya xuuSahitya Parishad in 1894, while Rabindranath Tagore and Navinchandra Sen were the vice presidents of the society




    1. Consider the statements regarding the Home Rule League.

    1. S. Subramaniya Ayer was on associate of Annie Besant.

    2. It started on behalf of the Indian National Congress.

    3. It changed its name to Swarajya Sabha.

    4. Its flag had the Union Flag on it.

    Which of the following are correct?

    (a) 1, 2 and 3

    (b) 2, 3 and 4

    (c) 1, 3 and 4

    (d) Only 1 and 2

    Answer.: ©

    Explanation:

    Sir Subbier Subramania Iyer KCIE was an Indian lawyer, jurist and freedom fighter who, along Annie Besant, founded the Home Rule Movement. The movement lasted around two years between 1916–1918 and is believed to have set the stage for the independence movement under the leadership of Annie Besant and Bal Gangadhar Tilak to the educated Engl class Indians. In 1920 All India Home Rule League changed its name to Swarajya Sabha. The Indian Home Rule movement was a movement in British India on the lines of the Irish Home Rule movement and other home rule movements. The movement years between 1916–1918 and is believed to have set the stage for the independence movement under the leadership of Annie Besant and Bal Gangadhar Tilak to the educated English speaking upper class Indians. In 1920 All India Home Rule League changed its name to Swarajya Sabha.  

    Flag:





    Indian & Odisha Geography – Physical, Social, Economic Geography of India & the World.


    1. Match List-I and List-II and select the correct answer using the code given below:

    List-I List-II

    a. Faulting 1. Yardangs

    b. Wave erosion 2. Kettle Hole

    c. Ice Scouring 3. Escarpment

    d. Wind Erosion 4. Stacks

    Codes:

    a b c d

    (a) 3 4 1 2

    (b) 4 3 1 2

    (c) 3 4 2 1

    (d) 4 3 2 1


    Answer: C 

    Explanation: 

    • Yardang, large area of soft, poorly consolidated rock and bedrock surfaces that have been extensively grooved, fluted, and pitted by wind erosion. The rock is eroded into alternating ridges and furrows essentially parallel to the dominant wind direction. 

    • A kettle is a depression or hole in an outwash plain formed by retreating glaciers or draining floodwaters. The kettles are formed as a result of blocks of dead ice left behind by retreating glaciers, which become surrounded by sediment deposited by meltwater streams as there is increased friction. 

    • An escarpment is a steep slope or long cliff that forms as a result of faulting or erosion and separates two relatively level areas having different elevations. The terms scarp and scarp face are often used interchangeably with escarpment 

    • A stack or sea stack is a geological landform consisting of a steep and often vertical column or columns of rock in the sea near a coast, formed by wave erosion.  


    1. Carefully read the following assertion(A) and reason(R) and select the correction from the codes:

    Assertion (A): Seandinavian countries have a regressive age-sex population structure

    Reason (R): The population growth rate is negative in these countries. For the selecting the correct answer, use the following code:

    (a) Both Assertion (A) and Reason (R) are true and Reason (R) is correct explanation of Assertion (A).

    (b) Both Assertion (A) and Reason (R) are true but Reason (R) is not a correct explanation of Assertion (A).

    (c) Assertion (A) is true and Reason (R) is false.

    (d) Assertion (A) is false and Reason (R) is true.


    Answer: C 

    Explanation: 

    Nordic countries are used unambiguously for Denmark, Norway, Sweden, Finland and Iceland, including their associated territories Greenland. A regressive population age structure is characterized by a relatively high proportion of elderly people and a low proportion of young people. This can be caused by low birth rates, high death rates, and increased life expectancy. It can be negative when death rate exceeds birth rate. This does not usually happen but in rare cases, it does happen. Again, this means more deaths and emigration, or the leaving of a country, than births and immigration, or entering a country. The regressive population has a low death rate and a declining birth rate. The pyramid has a narrow base and narrow top. Developed countries such as Sweden, France, Germany are good examples of regressive population. (0.2 ETC)


    1. Match List-I and List-II and select the correct answer using the code given below:

    List-I (Major cultural region of world) List-II (inhabitants)

    a. Arctic region 1. Eskimos

    b. Desert 2. The Indians, Negroes and Immigrants

    c. Latin-American Region 3. Samoans, Tahitian, Tongan, etc.

    d. Pacific Oceanic Region 4. Caravan

    Code:

    a b c d

    (a) 4 1 3 2

    (b) 1 4 2 3

    (c) 1 3 2 4

    (d) 4 2 3 1


    Answer: B 

    Explanation: 

    ESKIMO: a member of an indigenous people inhabiting northern Canada, Alaska, Greenland, and eastern Siberia, and traditionally living by hunting seals and other Arctic animals and birds and by fishing. Samoans or Samoan people are the indigenous Polynesian people of the Samoan Islands, an archipelago in Polynesia, who speak the Samoan language. A caravan or cafila is a group of people traveling together, often on a trade expedition. Caravans were used mainly in desert.  


    1. . Match List-I with List-II according to Koppen’s Climatic classification system and select the correct answer from the codes given:

    List-I (Climate Types) List-II (Characteristics)

    a. Af i. Humid Tropical Climate

    b. Aw ii. Tropical Humid and Dry climate

    c. Am iii. Monsoon Cliate

    d. As iv. Dry summers

    Codes:

    a b c d

    (a) i ii iii iv

    (b) ii iii I iv

    (c) iii i ii iv

    (d) iv ii I iii


    Answer: A

     Explanation:



    1. Given below are two statements, one labeled as Assertion (A) and the other labeled as Reason (R). Select your answer from the codes given below:

    Assertion (A): The Peru desert is recognized as a Cold Desert

    Reason (R): The temperature of Humbolt current plays a dominant role in controlling the surface atmospheric conditions of the coastal areas.

    (a) Both Assertion (A) and Reason (R) are true and Reason (R) is correct explanation of Assertion (A).

    (b) Both Assertion (A) and Reason (R) are true but Reason (R) is not a correct explanation of Assertion (A).

    (c) Assertion (A) is true and Reason (R) is false.

    (d) Assertion (A) is false and Reason (R) is true.

    Answer: A 

    Explanation: 

    The Sechura Desert is a coastal desert located south of the Piura Region of Peru along the Pacific Ocean coast and inland to the footh the Andes Mountains. Its extreme aridity is caused by the upwelling of cold coastal waters and subtropical atmospheric subsidence, but it is also subject to occasional flooding during El Niño years. 

    The Humboldt Current, also called the Peru Current, is a cold, low-salinity ocean current that flows north along the western coast of South America. It is an eastern boundary current flowing  in the direction of the equator, and extends 500– 1,000 km (310–620 mi) offshore. 

    The Humboldt Current is named after the German naturalist Alexander von Humboldt even though it was discovered by José de Acosta 250 years before Humboldt. The Humboldt has a considerable cooling influence on the climate of Chile, Peru and Ecuador. It is also largely responsible for the aridity of Atacama Desert in northern Chile and coastal areas of Peru and also of the aridity of southern Ecuador. Marine air is cooled by the current and thus is not conducive to generating precipitation (although clouds and fog are produced).  


    1. Match the following with respect to the occurrence of races in List-II to the type of Races in List-I.

    List-I Races List-II Area of Existence

    a. the Turko-Iranian 1. Rajasthan and Punjab

    b. The Indo-Aryan 2. Odisha and Assam

    c. The Seytho-Dravidian 3. West India (Gujarat to Karnataka)

    d. The Mongolo-Dravidian 4. North Western Frontier Agency

    Codes:

    a b c d

    (a) 4 1 3 2

    (b) 1 4 2 3

    (c) 1 3 2 4

    (d) 4 2 3 1


    Answer: A 

    Explanation: 

    There are 7 Main Types of Ethnological Groups in India 

    • The Turko-Iranian: The people of this type are found in North Western Frontier Province, Baluchistan and in the districts of Punjab and Sindh lying to the west of Indus. According to Risley these people are the mixture of Indian Aryans, Persian Aryans and Mongolo-Asiatic people. Their structure is above mean, complexion fair; eyes mostly dark, but occasionally grey; hair on face plentiful; head broad; nose moderately narrow, prominent and very long. 

    • The Indo-Aryan: The people of this type are mainly found in Kashmir, Punjab and Rajputana. Their structure is mostly tall, complexion fair, eyes dark; hair on face plentiful; head long; nose narrow and prominent but not specially long. 

    • The Scythio-Dravidian: People of this type are found in East Sindh Gujarat and Bombay. They can easily be distinguished from the people of Turko-Iranian type by lower structure, greater length of head, a higher nasal index, a shorter nose and a lower orbit nasal index. The prehistorical races made maximum contribution to this category of people. 

    • The Mongolo-Dravidian: The people of this type are found in Bengal, and Orissa. Their head is broad, complexion dark hair on face usually plentiful, stature medium, nose medium, with a tendency to broadness. 

    • The Dravidian: The people of this group are chiefly found in Southern Peninsula. Their stature is short or below mean; the complexion very dark, approaching black, hair plentiful with an occasional tendency to curl; eyes dark; head long; nose very broad; sometimes depressed at the root, but not so as to make the face appear flat. 

    • The Aryo-Dravidian: The people of this type are found in East Punjab, U.P. and Bihar. Their head form is long with a tendency to medium, the complexion varies from lightish brown to black, the nose ranges from medium to broad, being always broader than among the Indo-Aryans; the stature is lower than in the later group and usually below the average. 

    • The Mongoloid: This type is found in Burma; Assam and Sub-Himalayan tracts. The features of the people of this race are a broad head, dark complexion with a yellowish tinge; hair on face scanty; stature short or below average; nose broad; face characteristically flat; eyelids often oblique. These people are supposed to have come to India from the trans-Himalayan regions through the river valleys. 


    1. India is an agricultural country, where most of the population directly or indirectly involved in this sector. Regarding agriculture in India, which one of the following statements is/are not correct?

    1. Bajra is a Kharif crop, which requires 25o-31oC temperature and 40-50 cm rainfall.

    2. Cotton is categorized into three groups long-staple, medium staple and short-staple.

    3. Currently, India accounts for about 50% of the world’s total jute production.

    4. India is the second-largest producer of silk and the largest consumer of silk in the world.

    Select the correct answer using the codes given below:

    (a) 2 and 4

    (b) Only 3

    (c) 1 and 2

    (d) Only 4


    Answer: B 

    Explanation: 

    India is the leading jute goods-producing country in the world, accounting for about 70% of estimated world production. Hence, Statement 3 is incorrect. India is currently the world's second-largest producer of raw silk and the largest consumer of raw silk and silk fabrics. Statement 4 is correct. Bajra is a Kharif Crop. Temperature between 25- 30 degrees centigrade Rainfall of about 40 to 50 cm is best suitable for Bajra production. Statement 1 seems to be correct. In India, different types of cotton can be distinguished based on length, strength, and structure of the fibres, and on the basis of species. Based on the length, strength, and structure, they can be classified into long staple cotton, medium staple cotton, and short staple cotton. Whereas, based on species, they can be classified into Gossypium hirsutum (Upland Cotton), and Gossypium barbadense (Extra-Long Staple Cotton or Pima Cotton). With the adoption and spread of organic farming in India, now organic cotton also is grown widely in the country. Statement 2 is correct. 

    https://tractorkarvan.com/blog/types-of-cotton-india


    1. Match important projects in List-I with the area of implementation in List-II and select the correct answer using the code given below:

    List-I (Multipuropse) List-II (Area/Place of implementation)

    a. Tungabhadra Multipurpose Project 1. Lower Narmada Valley, Gujarat

    b. The Sardar Sarovar Project 2. Nalgonda District, Telangana

    c. Nagarajuna Sagar Project 3. Andhra Pradesh and Karnataka

    d. Bhakra Nagal Project 4. Punjab, Haryana and Rajasthan

    Codes:

    a b c d

    (a) 1 2 3 4

    (b) 2 3 1 4

    (c) 3 1 2 4

    (d) 4 2 1 3


    Answer: C 

    Explanation: 

    • The Tungabhadra Dam, also known as Pampa Sagar, is a water reservoir constructed across the Tungabhadra River in the city of Hosapete in Vijayanagara district, Karnataka. It is a multipurpose dam serving irrigation, electricity generation, flood control, etc. for the state. 

    • The Sardar Sarovar Dam is a concrete gravity dam built on the Narmada River in Navagam near the town of Kevadiya, Narmada District, in the state of Gujarat. The dam was constructed to provide water and electricity to four Indian states: Gujarat, Madhya Pradesh, Maharashtra, and Rajasthan. 

    • Nagarjuna Sagar Dam is a masonry dam across the Krishna River at Nagarjuna Sagar which straddles the border between Palnadu district in Andhra Pradesh and Nalgonda district in Telangana. 

    • Bhakra Nangal Dam is a concrete gravity dam on the Satluj River in Bhakra Village near Bilaspur in Bilaspur district, Himachal Pradesh in northern India. The dam forms the Gobind Sagar reservoir.


    https://www.iasgyan.in/daily-current-affairs/sardar-sarovar-dam


    https://www.iasgyan.in/daily-current-affairs/bhakra-dam


    https://www.iasgyan.in/blogs/dams-and-reservoirs-in-india


    https://www.indiatoday.in/education-today/gk-current-affairs/story/bhakra-nangal-dam-things-you-should-know-about-the-second-tallest-dam-in-asia-1372739-2018-10-22



    1. Among the following options which mountain pass doesn’t belongs to the correct state?

    (a) Shipki La → Himachal Pradesh

    (b) Aghil Pass → Uttarakhand

    (c) Dipher Pass → Arunachal Pradesh

    (d) Jelep La → Sikkim


    Answer: B Explanation: Aghil Pass is located in Ladakh, specifically in the Karakoram to the north of Mount Godwin Austin. It connects the Xinjiang province within China with Ladakh. Rest of the pairs are correct. 

    https://www.iasgyan.in/daily-current-affairs/jhelum-river


    https://www.iasgyan.in/daily-current-affairs/border-security-force-bsf


    https://www.iasgyan.in/daily-current-affairs/central-armed-police-forces


    https://www.iasgyan.in/daily-current-affairs/nathu-la


    https://de.wikipedia.org/wiki/Aghil-Pass



    1.  Which of the following factors are responsible for breaks in the monsoon?

    I. Origin of tropical cyclones in Bay of Bengal

    II. Erratic behavior of 2nd equatorial troughs

    III. Inadequate heating of Tibetan Plateau

    IV. Southern Branch of South West Tropical Jet re-establish over northern India

    Choose the correct options from below:

    Option: 

    (a) I and IV

    (b) I, II and III

    (c) I, II and IV

    (d) IV and III

    Answer: C 

    Explanation: 

    A “break in monsoon” refers to the brief period where the monsoon rains cease for a few days or weeks, a phenomenon commonly seen in the Indian subcontinent. It is characterized by days or a week or two of dry spells while the Monsoon season is ongoing, during which there is little or no rainfall. The break in the monsoon is caused by the shifting of the monsoon trough, an area of low pressure, which leads to a reduction in rainfall over certain areas. A break in the monsoon can impact agriculture due to reduced rainfall. It can also lead to floods in certain areas where the rainfall intensifies. Statements 1,2 and 4 are correct. https://cherrapunjee.com/movement-of-the-monsoon-trough-and-breaks-in-the-monsoon/


    1.  Which of the following river/rivers are exhibiting the antecedent drainage pattern in Odisha?

    (a) The Mahanadi and the Brahmani

    (b) The Brahmani and the Baitarini

    (c) The Mahanadi and the Baitarini

    (d) The Baitarini and the Rushikulya

    Ans.: (a)

    Explanation: 

    • An antecedent stream is a stream that maintains its original course and pattern despite the changes in underlying rock topography. Many Himalayan rivers are good examples of antecedent origin. These rivers originated well before the Himalayan region was uplifted. The rivers Indus, Brahmaputra, Sutlej, Kosi and Subansiri originated on the Tibetan side and now traverse the existing mountain ranges, cutting deep gorges. 

    • The Rivers which follow the general direction of slope are known as the consequent rivers. Most of the rivers of peninsular India are consequent rivers. For example, rivers like Godavari, Krishna and Kaveri, descending from the Western Ghats and flowing into the Bay of Bengal, are some of the consequent rivers of Peninsular India.  

    1.  Black Earth soil has been developed in which region of Odisha?

    (a) Brahmani Basin

    (b) Mayurbhanj 

    (c) Anugul – Athamallik and Boudh

    (d) Sambalpur Valley


    Answer: C 

    Explanation: 

    There are no regular occurrence of black soils in the state. These soils occur sporadically in the districts of Puri, Ganjam, Malkangiri, Kalahandi, Nuapada, Bolangir, Sonepur, Boudh, Sambalpur, Bargarh and Angul covering an area of 0.96 m. ha. of lands. The black colour of the soil is due to presence of titaniferous magnetite, humins, bitumins etc. These soils are formed due to weathering of basic rocks in the low lying areas 


    1. Match the morphological units of Rolling uplands (List-I) with the correct river basin (List-II)

    List-I (Morphological Units) List-II (Basin Associated)

    a. The Rajgangpur – Panposh Uplands 1. Ib Basin

    b. the Jharsuguda Uplands 2. Northern Tel Basin

    c. The Balagir – Titlagarh – Patnagarh Uplands 3. The Sabari Basin

    d. the Malakanagiri Uplands 4. Koel and the Sankh Rivers

    Codes:

    a b c d

    (a) 4 1 2 3

    (b) 1 2 3 4

    (c)   3 2 1 4

    (d) 2 3 1 4


    Answer: A 

    Explanation: 

    The high plateaus are found in mountain ranges with an average elevation of 300-600 meters. They are almost flat and the monotony of orography is interrupted by the deep river valleys. The plateaus are morphologically divided into the following divisions:- 

    a) The Panposh-Kendujhar-pala Lahara plateau of the upper Baitarani catchment basin b) The Nabaranyapur-Jaypur plateau of the upper Sabari basin 

    The rolling uplands are lower in elevation and they vary between 150 and 300 meters. These uplands are the product of revise action and are flat in nature. They are rich in soil nutrients and afford good opportunities for cultivation of paddy in wet areas. The rolling uplands are grouped into the following morphological units:- 

    a) The Rajgangpur-panposh uplands of the Koel and the Sankh rivers. 

    b) The Jharsuguda uplands of the Ib basin. 

    c) The Baragarh uplands of the Jira and the Jhaun basins. 

    d) The Balangir-Titilagarh-Patnagarh uplands of northern Tel basin 

    e) The Bhawanipatna uplands of the southern Tel basin 

    f) The Malkangiri uplands of the Sabari basin


    1. Which of the following is associated with Chilika lake?

    I. First Indian wetland under the Ramsar Convention.

    II. Irrawaddy dolphins

    III. Lacustrine plain

    Select the correct answer using the code given below:

    (a) I and II only

    (b) I and III only

    (c) I only

    (d) I, II and III


    Answer: D 

    Explanation: 

    • Chilika Lake is the largest brackish water lagoon in Asia and the second-largest coastal lagoon in the world. It is one of the hotspots of biodiversity in the country. It is spread over the Puri, Khordha, and Ganjam districts of Odisha at the mouth of the Daya River, flowing into the Bay of Bengal. 

    • The north shore of the lake is part of Khordha District and the western shore is part of Ganjam District. Chilika Lake has been listed as a tentative UNESCO World Heritage site. In 1981, Chilika Lake was designated the first Indian wetland of international importance under the Ramsar Convention. 
    • It is the largest wintering ground for migratory birds on the Indian sub-continent. Birds from as far as the Caspian Sea, Lake Baikal, Aral Sea, and other remote parts of Russia, Kirghiz steppes of Kazakhstan, Central and Southeast Asia, Ladakh, and the Himalayas come here. 

    • The lake is home to a number of threatened species of plants and animals. Around 152 rare and endangered Irrawaddy dolphins have also been reported. 

    • The rare and threatened animal species identified are green sea turtle (EN), dugong (VU), Irrawaddy dolphin (EN), blackbuck (NT), Spoon billed sandpiper (CR), Chilika limbless skink (CR) and fishing cat (VU). South coastal plain of Odisha is the Lacustrine plain of Chilika Lake and the Rushikulya river delta.  


    https://www.iasgyan.in/daily-current-affairs/chilka-lake


    https://www.gktoday.in/question/a-plain-formed-due-to-the-past-existence-of-lake-a


    1. Odisha experiences which types of climates according to Koppen’s climatic classification?

    (a) Tropical Savanna

    (b) Humid Subtropical

    (c) Mediterranean 

    (d) Subtropical steppe

    Answer: A 

    Explanation: 

    Odisha has a tropical climate, characterized by high temperatures, high humidity, medium to high rainfall, and short and mild winters. As per Thornthwaite’s classification, Odisha comes under the Subhumid category, implying deficient winter rains. As per Koppen’s climatic classifications, most part of Odisha comes under the AW having a tropical Savannah type of climate.  


    Indian polity and governance-Constitution, political system, Panchayati raj, public policy, Rights Issues, etc.



    1. Which of the following statements are correct in respect of ‘Objective Resolution’ of Indian Constitution?

    (1) The Author of this Resolution was Dr. B.R. Ambedkar

    (2) It was adopted by Constituent Assembly on January 22, 1947

    (3) The goal of this Resolution was to declare India as a Sovereign, Socialist, Democratic Republic

    (4) This Resolution later framed as the Preamble of Indian Constitution

    (a) 1 and 3

    (b) 2 and 4

    (c) 1 and 4

    (d) 1 and 2


    Answer: B 

    Explanation: 

    The Objective Resolution was moved on December 13, 1946 by Pandit Jawaharlal Nehru,which provided the philosophy and guiding principles for framing the Constitution and later took the form of Preamble of the Constitution of India. This Resolution was unanimously adopted by the Constituent Assembly on 22 January 1947. It forms the basis of Preamble of Indian Constitution. So statements 2 and 4 are correct. 


    https://www.iasgyan.in/blogs/the-indian-constitution-important-things-you-should-know


    1. Which of the following statements are correct in respect of ‘Constituent Assembly’?

    (1) The Constituent Assembly was constituted under the scheme of ‘Cabinet Mission Plan’

    (2) The total strength of Constituent Assembly originally was 369

    (3) 296 seats were allotted to British India

    (4) 73 seats to Princely States

    (a) 2 and 3

    (b) 3 and 4

    (c) 1 and 4

    (d) 1 and 3

    Answer: D 

    Explanation: The Cabinet Mission plan aimed to make a Constituent Assembly of India and avoid the partition. The total strength of the Constituent Assembly was to be 389. Of these, 296 seats were to be allotted to British India and 93 seats to the princely states. Statements 2 and 4 are incorrect


    1. Consider the following pairs in respect of ‘Sources of Indian Constitution’ and pick up the right matchings. 

    Item Source 

    (1) Office of Governor - England 

    (2) Public Service Commission - Govt. of India 

    (3) Freedom of Trade & Commerce - USA 

    (4) The Language of Preamble of Constitution - Australia 

    (A) 1 and 2 

    (B) 2 and 3 

    (C) 2 and 4 

    (D) 3 and 4  


    Answer: C 

    Explanation:  

    Australia 

    o Concurrent list o Freedom of trade, commerce and intercourse 

    o Joint-sitting of the two Houses of Parliament 

     Canada 

    o Federation with a strong Centre 

    o Vesting of residuary powers in the Centre 

    o Appointment of state governors by the Centre 

    o Advisory jurisdiction of the Supreme Court 

     Ireland 

    o Directive Principles of State Policy o Nomination of members to Rajya Sabha 

    o Method of election of the president  Japan o Procedure Established by law  

    Soviet Union (USSR) (now, Russia) 

    o Fundamental duties 

    o Ideals of justice (social, economic and political) in the Preamble  

    UK 

    o Parliamentary government 

    o Rule of Law 

    o Legislative procedure 

    o Single Citizenship 

    o Cabinet system 

    o Prerogative writs 

    o Parliamentary privileges 

    o Bicameralism  

    US 

    o Fundamental rights 

    o Independence of judiciary 

    o Judicial review 

    o Impeachment of the president 

    o Removal of Supreme Court and High Court judges 

    o Post of vice-president  

    Germany (Weimar) 

    o Suspension of Fundamental Rights during emergency  

    South Africa 

    o Procedure for amendment in the Indian Constitution o Election of members of Rajya Sabha  

    France 

    o Republic

    https://en.wikipedia.org/wiki/Constitution_of_India#:~:text=The%20constitution%20was%20drawn%20from,the%20Indian%20Independence%20Act%201947


    1. Which of the following Articles of Indian Constitution are not related to each other? 

    (1) Art. 12 and Art. 36 

    (2) Art. 13 and Art. 368 

    (3) Art. 22 and Art. 32(2) 

    (4) Art. 141 and Art. 214 


    (A) 2 and 3 

    (B) 4 only 

    (C) 1 and 3 

    (D) All are related


    Answer: B 

    Explanation: Article 12 is related to fundamental rights and one can directly reach the court in case of its violation, whereas, article 36 is related to the directive principles of the state policy. Article 13 prevents the passing of laws which "take away or abridge" the Fundamental Rights provisions. Article 368 does not contain a power to amend the constitution but only a procedure. The power to amend comes from the normal legislative power of Parliament. Article 22 deals with the protection against arrest and detention in certain cases. This article is applicable to both citizens and non-citizens. This provision extends certain procedural safeguards for individuals in case of an arrest. The idea behind this right is to prevent arbitrary arrests and detention. Article 32, Constitution of India 1950 The Supreme Court shall have power to issue directions or orders or writs, including writs in the nature of habeas corpus, mandamus, prohibition,  quo warranto and certiorari, whichever may be appropriate, for the enforcement of any of the rights conferred by this Part. Article 141 provides that the law declared by the Supreme Court shall be binding on all courts within the territory of India. The law declared has to be construed as a principle of law that emanates from a judgment, or an interpretation of law or judgment by the Supreme Court, upon which, the case is decided. According to the Article 214 every State has a High Court operating within its territorial jurisdiction. Parliament has the power to establish a common High Court for two or more States.  


    https://www.constitutionofindia.net/articles/article-214-high-courts-for-states/#:~:text=Article%20214%2C%20Constitutio


    1. Which of the following statements are correct in respect of Lame Duck Politician?

    (1) Having lost a re-election bid

    (2) Choosing not to seek another term after expiration of his/her term.

    (3) No term limit running for particular office again

    (4) Abolition of Office after his/her official term

    (a) 1, 2 and 4

    (b) 2, 3 and 4

    (c) None of the above

    (d) All above


    Answer: A 

    Explanation: In politics, a lame duck or outgoing politician is an elected official whose successor has already been elected or will be soon. An outgoing politician is often seen as having less influence with other politicians due to their limited time left in office. Conversely, a lame duck is free to make decisions that exercise the standard powers with little fear of consequence, such as issuing executive orders, pardons, or other controversial edicts. Lame duck politicians result from term limits, planned retirement, or electoral losses, and are especially noticeable where political systems build in a delay between the announcement of results and the taking of office by election winners. Even at the local level, politicians who do not seek re-election can lose credibility and influence. Uncompleted projects may fall to the wayside as their influence diminishes. 



    The status can be due to: 

    o having lost a re-election bid 

    o choosing not to seek another term, which would start at the expiration of the current term 

    o a term limit which prevents the official from running for that particular office again 

    o the abolition of the office, which must nonetheless be served out until the end of the official's term. 


    https://en.wikipedia.org/wiki/Lame_duck_(politics)


    1. Which of the following statements are correct in respect of Basic Structure of Indian Constitution?

    (1) It is a form of Judicial Review

    (2) It is used to test the legality of any legislation by the Courts

    (3) The Doctrine of Basic Structure of Constitution gave free hand to Parliament to amend any part of Constitution

    (4) The Constitution of India defines ‘basic structure’ in terms of Federalism, Secularism, Fundamental Rights and Socialism.

    (a) 1 and 2

    (b) 2 and 3

    (c) 1 and 4

    (d) 3 and 4


    Answer: A 

    Explanation: The Doctrine of Basic Structure is a form of judicial review that is used to test the legality of any legislation by the courts. The doctrine was evolved by the Supreme Court in the 1973 landmark ruling in Kesavananda Bharati v State of Kerala. In a 7-6 verdict, a 13- judge Constitution Bench ruled that the ‘basic structure’ of the Constitution is inviolable, and could not be amended by Parliament. 

    If a law is found to “damage or destroy” the “basic features of the Constitution”, the Court declares it unconstitutional. The test is applied to constitutional amendments to ensure the amendment does not dilute the fundamentals of the Constitutional itself. The test is widely regarded as a check on majoritarian impulses of the Parliament since it places substantive limits on the power to amend the Constitution. The Kesavananda ruling was a culmination of a series of tussles between the judiciary and the executive led by then Prime Minister Indira Gandhi. After a reversal of key legislation including land reforms; nationalization of banks; abolition of privy purse- the Parliament brought in a constitutional amendment to give itself the power to amend any part of the Constitution and passed a law that it cannot be reviewed by the courts. The Court had to then examine the scope of the Parliament’s power to amend the Constitution and the legality of the land reforms. The 13-judge bench gave 11 separate judgments and the doctrine was culled out as the majority opinion in the case. The Court ruled that while Parliament has vast powers to amend the Constitution, it cannot amend certain “basic features.” On land reforms, the Court upheld the amendment that removed the fundamental right to property. The court ruled that in spirit, the amendment would not violate the “basic structure” of the Constitution.  

    The origins of the basic structure doctrine are found in the post-war German Constitution law which, after the Nazi regime, was amended to protect some basic laws. In the Kesavananda ruling, the Supreme Court cited several aspects of the Constitution that could be identified as “basic features” of the document but added that it was not an exhaustive list. For example, judicial review, rule of law, federalism, and democratic republic structure are identified as basic features. In the 2015 ruling where the Supreme Court struck down the National Judicial Appointments Commission Act and the related Constitutional Amendment, “judicial independence” was identified as a basic feature of the Constitution. 

    Statements 3 and 4 are incorrect. Basic Structure dies not give free hand to amend any part of Constitution. Also, the Constitution does not define the Basic Structure. 


    1. Entry 5 of list II of Seventh Schedule of Indian Constitution deals with - 

    (a) Public Order

    (b) Public Health

    (c) State Pensions

    (d) Local Governments


    Answer: D 

    Explanation: The Seventh Schedule to the Constitution of India specifies the allocation of powers and functions between the Union and the State legislatures. It embodies three lists; namely, the Union List, the State List, and the Concurrent List. 

    Seventh Schedule List II—State List 

    1. Public order (but not including 3 [the use of any naval, military or air force or any other armed force of the Union or of any other force subject to the control of the Union or of any contingent or unit thereof] in aid of the civil power). 

    2. Police (including railway and village police) subject to the provisions of entry 2A of List I.] 

    3.. ***Officers and servants of the High Court; procedure in rent and revenue courts; fees taken in all courts except the Supreme Court. 

    4. Prisons, reformatories, Borstal institutions and other institutions of a like nature, and persons detained therein; arrangements with other States for the use of prisons and other institutions. 

    5. Local government, that is to say, the constitution and powers of municipal corporations, improvement trusts, districts boards, mining settlement authorities and other local authorities for the purpose of local selfgovernment or village administration. 6. Public health and sanitation; hospitals and dispensaries. 

    7. Pilgrimages, other than pilgrimages to places outside India. 

    8. Intoxicating liquors, that is to say, the production, manufacture, possession, transport, purchase and sale of intoxicating liquors. 

    9. Relief of the disabled and unemployable. 

    10. Burials and burial grounds; cremations and cremation grounds.  

    https://www.mea.gov.in/Images/pdf1/S7.pdf


    1. Which of the following statements are correct in respect of Art. 243 of Indian Constitution?

    (1) Art. 243 (C) provides representation of chairpersons of Panchayats at the Village level in the Panchayats at the District level.

    (2) Art. 243 (D) provides representation for women to the offices of chairperson in Panchayats at village level.

    (3) Art. 243 (I) is related to power to levy taxes by Panchayats.

    (4) Art. 243 (J) is related to bar on the courts to call in question the election of Panchayats.

    (a) 2 and 3

    (b) 1 and 4

    (c) 1 and 2

    (d) 1 and 3


    Answer: C 

    Explanation: Article 243 (C) of the Indian constitution deals with the Composition of Panchayats. The State Legislature has the power to make provisions for the Composition of Panchayats. All the seats in all the levels of Panchayati Raj shall be filled with the direct election. 

    Article 243D of the constitution under Part IX provides for reservation of seats for SC & ST in every Panchayat i.e. at all three levels. State Legislature shall provide for reservation of offices of chairperson in Panchayats for SCs & STs. Governor of a state constitutes (Under Article 243- I) a finance commission every five years which recommends the governor about the principles that govern the distribution of taxes, fees, tolls, and duties (collected by the state government) between the state and its PRIs at three levels. Article 243J of the Indian Constitution is related with the audit of the accounts of Panchayats. According to article 243J, "The Legislature of a State may, by law, make provisions with respect to the maintenance of accounts by the Panchayats and the auditing of such accounts".  

    https://www.constitutionofindia.net/articles/article-243-definitions/

    https://indiankanoon.org/doc/452231/


    1. Which of the following statements are correct in respect of the Election Commission of India?

    (1) It is an autonomous Legal Body

    (2) Part XV of Indian Constitution deals with the Elections and the Election Commission

    (3) The Election Commission was established on January 25, 1950

    (4) Art. 328 provides bar on the interference of Courts in electoral matters.

    (a) 2 and 3

    (b) 1 and 4

    (c) 3 and 4

    (d) 2 and 3


    Answer: A 

    Explanation: The Election Commission of India (ECI) is a constitutional body. It was established by the Constitution of India to conduct and regulate elections in the country. Part XV of the Constitution of India consists of Articles on Elections. Article 324 of the Constitution provides that the power of superintendence, direction, and control of elections to parliament, state legislatures, the office of the president of India, and the office of vice-president of India shall be vested in the election commission. Thus, the Election Commission is an all-India body in the sense that it is common to both the Central government and the state governments. Formed: 25 January 1950 Article 328: Power of Legislature of a State to make provision with respect to elections to such Legislature. Article 329: Bar to interference by courts in electoral matters.  

    https://www.constitutionofindia.net/articles/article-328-power-of-legislature-of-a-state-to-make-provision-with-respect-to-elections-to-such-legislature/#:~:text=Article%20328%3A%20Power


    1. Which of the following Committees recommended for the creation of ‘Nyaya Panchayats’ at village level? 

    (A) L.M. Singhvi Committee 

    (B) G.V.K. Rao Committee 

    (C) Mohan Kanda Committee 

    (D) Ashok Mehta Committee  


    Answer: D 

    Explanation: 

    ASHOK MEHTA COMMITTEE  

    • The 3-tier system of Panchayati Raj should be replaced by the 2-tier system: Zilla Parishad at the district level, and below it, the Mandal Panchayat consisting of a group of villages covering a population of 15000 to 20000.  

    • A district should be the first point for decentralisation under popular supervision below the state level.  

    • Zila Parishad should be the executive body and made responsible for planning at the district level.  

    • There should be an official participation of political parties at all levels of Panchayat elections.  

    • The Panchayat Raj institutions should have compulsory powers of taxation to mobilise their own financial resources.  

    • There should be a regular social audit by a district level agency and by a committee of legislators to check whether the funds allotted for the vulnerable social and economic groups are actually spent on them.  

    • The state government should not supersede the Panchayat Raj institutions. In case of an imperative supersession, election should be held within 6 months from the date of supersession.  

    • The Nyaya Panchayats should be kept as separate bodies from that of development of the state in consultation with the Chief Election Commissioner should organise and conduct the Panchayati Raj elections.  

    • Development functions should be transferred to the Zila Parishad and all development staff should work under its control and supervision.  

    • The voluntary agencies should play an important role in mobilising the support of the people for Panchayati Raj.  

    • A minister for Panchayati Raj should be appointed in the state council of ministers to look after the affairs of the Panchayati Raj institutions

    1. Which of the following statements are correct in respect of ‘Attorney General of India?

    (1) Attorney General of India is a part of India Judiciary

    (2) Attorney General of India is appointed by President on the advice of Chief Justice of India

    (3) The term of office of Attorney General of India is not fixed by the Constitution

    (4) Attorney General of India has right to take part in the proceedings of both Houses of Parliament.

    (a) 1, 2 and 4

    (b) 2 and 3

    (c) 1 and 3

    (d) 3 and 4


    Answer: D 

    Explanation: 

    Statement 3 and 4 are correct: The Indian constitution under Article 76 has provided for the office of the Attorney General for India. He is the highest law officer in the country. He is appointed by the President of India. He must be a person who is qualified to be appointed a judge of the Supreme Court. The term of office of the AG is not fixed by the constitution. Further, the constitution does not contain the procedure and grounds for his removal. He holds office during the pleasure of the president. This means that he may be removed by the president at any time. He may also quit his office by submitting his resignation to the president. Conventionally, he resigns when the government (council of ministers) resigns or is replaced, as he is appointed on its advice. The remuneration of the AG is not fixed by the constitution. He receives such remuneration as the President may determine. The Attorney General is not a full-time counsel for the Government. He does not fall into the category of a government servant. Further, he is not debarred from private legal practice.

    https://www.iasgyan.in/daily-current-affairs/attorney-general


    1. Which of the following statements are correct in respect of Bills introduced in Parliament?

    (1) If a Government Bill is rejected by the House in Parliament it amounts ‘vote of confidence’ in the Government

    (2) The Government’s Bill introduction in the House necessitates a fifteen-day notice

    (3) The ‘Taxation Laws (Amendment) Bill 2021’ comes under the category of Money Bill

    (4) The Central Universities (Amendment) Bill 2021 comes under Ordinary Bill.

    (a) 1 and 4

    (b) 2 and 3

    (c) 3 and 4

    (d) 2 and 4


    Answer: C 

    Explanation: Statement 1 and 2 are incorrect: In parliamentary systems, the rejection of a government bill does not necessarily amount to a vote of confidence in the government. In fact, the rejection of a specific bill does not automatically result in the fall of the government. In the Indian parliamentary system, the introduction of a government bill does not necessarily require a specific notice period of 15 days. However, there is a general practice and procedure regarding the giving of notice before the introduction of bills in the Parliament


    1. Which of the following statements are correct in respect of ‘Types of Majority in Parliament’?

    (1) Absolute majority-More than 50% of the House’s total Strength

    (2) Simple Majority-The majority of more than 50% of those present and voting

    (3) Effective Majority: Majority of the House’s effective strength.

    (4) Effective Strength: means total strength excluding number of vacancies

    (5) Special Majority: Majority of 2/3 members of total strength of the House

    (a) 1, 3, 4 and 5

    (b) 2, 3 and 5

    (c) 2, 3 and 4

    (d) 1, 2, 3 and 4


    Answer: D 

    Explanation: Simple This refers to a majority of more than 50% of the members present and voting in the House. Also known as Functional or Working majority, this is the most commonly used type of majority. When the law does not specify the kind of majority needed, a simple majority is used for passing bills or motions. Absolute This refers to a majority of more than 50% of the House's total membership. Effective This refers to a majority of more than 50% of the effective strength of the House. In the Constitution, an effective majority is mentioned as “all the then members” Special Any majority other than simple, absolute and effective are called Special Majorities. Special Majority as Per Article 249: requires a majority of 2/3rd members present and voting. To pass the Rajya Sabha resolution to empower the parliament to make laws in the state list. Special Majority as Per Article 249: requires a majority of 2/3rd members present and voting. To pass the Rajya Sabha resolution to empower the parliament to make laws in the state list. Special Majority as Per Article 61: requires a majority of 2/3rd members of the total strength of the house. For the impeachment of the Indian President. Special Majority as Per Article 368: requires a majority of 2/3rd members present and voting supported by more than 50% of the total strength of the house. This type of majority is used for most of the Constitutional amendment bills. Cases where special majority as per article 368 is used: To pass a constitutional amendment bill which does not affect federalism. Removal of judges of SC/HC. Removal of CEC/CAG. Approval of national emergency requires special majority as per Article 368 in both houses. Resolution by the state legislature for the creation/abolition of Legislative Council.


    1. Which of the following statements are correct in respect of ‘Sitting of Parliament’?

    (1) In the year 1955 a Lok Sabha Committee had proposed a time-table for parliamentary sessions with three times a year

    (2) It suggested Budget Session from February 1, to May 7

    (3) It suggested Monsoon Session from July 1 to September 10

    (4) It suggested Winter Session from November 15 to December 15

    (a) 1 and 4

    (b) 2 and 3

    (c) 1 and 2

    (d) 3 and 4


    Answer: C 

    Explanation: Only statement 1 and 2 are correct: India’s Parliament has no fixed calendar of sittings. In 1955, a Lok Sabha committee had proposed a timetable for parliamentary sessions. It recommended that the Budget session of Parliament begin on February 1 and go on till May 7, and the Monsoon session start on July 15 and end on September 15. The committee suggested that the Winter session, the last session of the year, commence on November 5 (or the fourth day after Diwali, whichever is later) and finish on December 22. While the government agreed to this calendar, it was never implemented.

    https://indianexpress.com/article/explained/explained-politics/special-session-of-parliament-8920401/


    1. Which of the following statements are correct in respect of ‘Procedure established by Law’ and ‘Due process of Law’?

    (I) Under the principle of ‘Procedure established by Law’, as long as the government follows the procedures it has established, it’s actions are considered valid, even if they may appear unfair, or unjust to individuals

    (II) ‘Due Process of Law’ doctrine not only checks if there is a Law to deprive the life and personal liberty of a person but also sees if the Law made is fair, just and not arbitrar4y.

    (III) ‘Procedure established by Law’ is an American doctrine and ‘Due Process of Law’ is Indian constitutional doctrine.

    (a) I is correct and II and III is not correct

    (b) I and II are correct and III is not correct

    (c) I and III are correct and II is not correct

    (d) I, II and III are correct


    Answer: B 

    Explanation: Statement 3 is incorrect: Due process of lawOriginated from the Constitution of the United States. Article 21 of the Indian Constitution mentions the phrase procedure established by law. The Indian Constitution does not explicitly mention the phrase due process of law.

    https://blog.ipleaders.in/procedure-established-by-law/#:~:text=Due%20process%20of%20law,-Meaning&text=Originated%20from%20the%20Co


    1. Which of the following while pursuing common interests of their Groups, attempt to influence the Public Policy making?

    (a) Pressure Groups

    (b) Elite Groups

    (c) Bureaucracy

    (d) Political Parties

    Answer: A 

    Explanation: The pressure group is defined as a special interest group which seeks to influence Government policy in a particular direction; action groups are loosely organized pressure groups. Such groups do not seek Government control or responsibility for policy, and their political function is not officially recognized. Pressure groups may be criticized for pressing interests not in keeping with general welfare, for using unacceptable pressure techniques.

    https://www.ojp.gov/ncjrs/virtual-library/abstracts/police-and-pressure-groups


    1. During which part of the ‘public policy cycle’ does the government explore potential policy responses?

    (a) Values setting

    (b) Policy formulation

    (c) Policy implementation

    (d) Policy feedback


    Answer: B. 

    Explanation: The government explores potential policy responses during the "Policy Formulation" stage of the public policy cycle. This stage involves the development and evaluation of different policy options to address a particular issue or problem. It includes conducting research, analyzing data, consulting with experts and stakeholders, and considering various factors such as feasibility, effectiveness, and cost. Once the potential policy responses are explored and formulated, they can then be considered for implementation in the next stage of the policy cycle.  

    https://www.egyankosh.ac.in/bitstream/123456789/76653/1/Unit-3.pdf


    1. The final six months of the Election cycle, when the government announces popular government initiatives is known as -

    (a) Green Z

    (b) Yellow Zone

    (c) Red Zone

    (d) Blue Zone


    1. Which of the following are the features of ‘Good Governance’?

    (1) Accountability 

    (2) Conservatism 

    (3) Rigidity 

    (4) Transparency 

    (5) Integrity 

    (6) Micro Management

    (a) 2, 3 and 6

    (b) 1, 2 and 6

    (c) 3, 4 and 5

    (d) 1, 4 and 5


    Answer: D 

    Explanation: Eight principles of good governance Citing from the United Nations Economic and Social Commission for Asia and the Pacific (UNESCAP), the concept of good governance has eight principles. 

    1. Participation Participation in the concept of good governance here is an opportunity for everyone to voice their opinions through institutions or representations. In addition, everyone, without exception, has the right to freedom of association and expression. 

    2. Rule of law To implement good governance, the legal framework in the country must be enforced impartially, especially concerning human rights law. 

    3. Transparency Transparency means that every policy taken and implemented by the government must be carried out under existing regulations. In addition, there must be a guarantee that any information related to the policy can be accessed by everyone, especially those who are directly affected by the policy. 

    4. Responsiveness Good governance needs institutions and processes to attempt to serve all stakeholders within a reasonable time. 

    5. Consensus oriented This fifth principle is related to the decisionmaking process. When the decision-making process cannot accommodate everyone’s wishes, then at a minimum, the decision must be a decision that can be accepted by everyone and does not harm anyone. 

    6. Equity and inclusiveness Good governance ensures justice for the community. Everyone has the same opportunity to maintain and improve their welfare. 

    7. Effectiveness and efficiency Every decision-making process and its institutions must be able to produce decisions that meet every community need. Community resources must also be utilised optimally by the government. 

    8. Accountability All institutions involved in good governance have full responsibility to the public for the sake of improving the quality of society.  

    https://uclg-aspac.org/good-governance-definition-and-characteristics/

    1. Which of the following are true about Public Policy?

    (1) It is a way of doing things including rules and regulations

    (2) It is a means of ‘authoritative allocation of values’

    (3) It is for Public Sector Organisations

    (4) ‘What the Government choose to do or not to do’

    (a) 1, 3 and 4

    (b) 1, 2 and 3

    (c) 2, 3 and 4

    (d) All above


    Answer: D 

    Explanation: Public policy is purposeful and goal-oriented, aiming to address specific issues, achieve objectives, or solve societal problems. Public policy is the result of governmental decisions and actions. It involves the use of authority, resources, and regulations to influence or direct behavior. The formulation of public policy involves a systematic decision-making process, which includes problem identification, agenda-setting, policy formulation, implementation, and evaluation. Public policy is designed to serve the public interest, reflecting the collective needs and welfare of the population. David Easton defines policy as the 'outputs' of the political system and public policy as 'the authoritative allocation of values for the whole society”. 

    https://www.egyankosh.ac.in/bitstream/123456789/76653/1/Unit-3.pdf


    1. Which one of the following statements is correct in respect of Art.21 of Indian Constitution?

    (1) It guarantees Right to Life and Personal Liberty

    (2) In the case of Narmada Bachao Andolan Vs Union of India 2020, Supreme Court held that ‘Right to Water’ is not a Fundamental Right but Legal Right

    (3) Right to water is inferred from the Right to Life

    (4) The Supreme Court also held that access to Clear Water is a Natural Right

    (a) 1 and 3

    (b) 2 and 4

    (c) 2 and 3

    (d) 1 and 4

    Answer: A 

    Explanation: 

    Statements 1 and 3 are correct: Water is the basic need for the survival of human beings and is part of the right to life and human rights as enshrined in Article 21 of the Constitution of India....and the right to healthy environment and to sustainable development are fundamental human rights implicit in the right to “life”. 

    https://nhrc.nic.in/sites/default/files/Right%20to%20water.pdf


    1. From which Constitution ‘Suspension of Fundamental Rights during Emergency’ was borrowed to Indian Constitution?

    (a) Soviet Constitution, USSR

    (b) Weimer Constitution of Germany

    (c) French Constitution

    (d) Swiss Constitution


    Answer: B 

    Explanation: The suspension of fundamental rights during emergencies in the Indian Constitution was inspired by the Weimar Constitution of Germany. The idea was borrowed from the German Constitution, which allowed for the suspension of fundamental rights during a state of emergency. The relevant provisions can be found in Article 359 of the Indian Constitution, which grants the President the power to suspend the enforcement of certain fundamental rights during a national emergency.  

    https://www.legalservicesindia.com/article/589/Position-of-Fundamental-Rights-duringEmergency.html


    1. Which of the following statements are correct in respect of ‘Right to Education’?

    (1) Art.21(A) of Indian Constitution provides Right to Education for free and compulsory education to all

    (2) Right to Education Act was passed by Indian Parliament on August 4, 2009

    (3) Right to Education came into force on November 14, 2009

    (4) Art.21(A) was inserted in the Constitution by 86th Constitutional Amendment Act, 2004

    (a) 1 and 3

    (b) 1 and 2

    (c) 1 and 4

    (d) 2 and 4

    Answer: B. 

    Explanation: Statements 1 and 2 are correct: The Right of Children to Free and Compulsory Education Act or Right to Education Act (RTE), is an Act of the Parliament of India enacted on 4 August 2009, which describes the modalities of the importance of free and compulsory education for children between 6 and 14 in India under Article 21a of the Indian Constitution. India became one of 135 countries to make education a fundamental right of every child when the Act came into force on 1 April 2010. The Right to Education (RTE) Act, which came into force in India, was enacted on April 1, 2010. The Constitution (Eighty-sixth Amendment) Act, 2002 inserted Article 21A in the Constitution of India. Its aim is to provide free and compulsory education of all children in the age group of six to fourteen years as a Fundamental Right.

    https://righttoeducation.in/know-yourrte/about#:~:text=The%20Right%20of%20Children%20to,21a%20of%20the%20Indian%20Constitution


    1. Which of the following are related to ‘Right to Privacy’?

    (1) Art.21 of Indian Constitution

    (2) Art.12 of Universal Declaration of Human Rights

    (3) Art.17 of the International Covenant on Civil and Political Rights.

    (4) Art.8 of European Convention on Human Rights

    (a) 1, 3 and 4

    (b) 1, 2 and 4

    (c) 2, 3 and 4

    (d) All above

    Answer: D. 

    Explanation: Article 17 No one shall be subjected to arbitrary or unlawful interference with his privacy, family, home or correspondence, nor to unlawful attacks on his honour and reputation. Article 12 of the Universal Declaration of Human Rights states: No one shall be subjected to arbitrary interference with his privacy, family, home or correspondence, nor to attacks upon his honour and reputation. Everyone has the right to the protection of the law against such interference or attacks ARTICLEEveryone has the right to respect for his private and family life, his home and his correspondence. 

    https://www.ohchr.org/en/instruments-mechanisms/instruments/international-covenant-civil-and-political-rights#:~:text=Article%2017,-1.&text=No%20one%20shall%20be%20subjected


    1. Which Article of Universal Declaration of Human Rights tells that the ‘Right of Nationality’ depends on one’s wish. “Every one has the Right to a Nationality”?

    (a) Article 15

    (b) Article 20

    (c) Article 25

    (d) Article 30


    Answer: A. 

    Explanation: Article 15 of the 1948 Universal Declaration of Human Rights, "Everyone has the right to a nationality. No one shall be arbitrarily deprived of his nationality nor denied the right to change his nationality."

    https://www.ohchr.org/en/nationality-and-statelessness/international-standards-relating-nationality-and-statelessness#:~:text=Article%2015%20of%20the

     

    General issues on Environmental Ecology, Bio-diversity, and climate change that don't require subject specialization.



    1. Gases that trap heat in the atmosphere and cause global warming are called greenhouse bases.

    Which of the following combinations are not included under the greenhouse gases?

    (a) Carbon dioxide, Methane

    (b) Nitrous oxide, Hydrofluorocarbons

    (c) Carbon monoxide, Sulphur dioxide

    (d) Carbon dioxide, water vapour


    Answer: C 

    Explanation: 

    • Carbon Dioxide (CO2): A major greenhouse gas produced through the combustion of fossil fuels and other human activities, contributing to global warming and climate change. 

    • Methane (CH4): A potent greenhouse gas released during the production and transport of coal, oil, and natural gas. It has a higher warming potential than carbon dioxide over a short time frame. 

    • Nitrous Oxide (N2O): Emitted from agricultural and industrial activities, as well as the burning of fossil fuels and solid waste. It contributes to both climate change and ozone layer depletion. 

    • Ozone (O3): While ozone in the stratosphere protects life on Earth by blocking ultraviolet radiation, groundlevel ozone is a greenhouse gas and air pollutant formed by chemical reactions between oxides of nitrogen and volatile organic compounds. 

    • Water Vapour (H2O): The most abundant greenhouse gas in the Earth's atmosphere. While human activities do not directly control water vapour levels, it is a significant contributor to the natural greenhouse effect."  

    https://www.eia.gov/energyexplained/energy-and-the-environment/greenhouse-gases.php


    1. More than 90 percent of all nitrogen are fixed as ammonia, nitrites, and nitrates by soil microorganisms. Symbiotic bacteria associated with leguminous plants fix nitrogen by the formation of root nodules. In which of the following plant nitrogen fixation does not occurs?

    (a) Common Beans

    (b) Soybeans 

    (c) Peanuts 

    (d) Oats

    Answer: D

    Explanation: Oats undergo C3 photosynthesis, which is a type of carbon fixation. In C3 photosynthesis, the primary carbon fixation process occurs in the mesophyll cells of the plant's leaves. The enzyme responsible for carbon fixation in C3 plants is ribulose-1,5-bisphosphate carboxylase/oxygenase (RuBisCO). Hence option D is incorrect.  

    https://www.treehugger.com/how-nitrogen-fixing-plants-can-perk-your-garden-4863746#:~:text=Cereal%20grains%20such%20a


    https://eos.com/blog/nitrogen-fixation/#:~:text=The%20list%20of%20nitrogen%2


    1. For protection of plants and animal species and to provide for the prevention and control of water pollution, air pollution and the environment, government of India has taken adequate steps and several prevention, control and protection Acts has been enacted.

    Which of the following combination of statements is correct?

    (a) The water (Prevention and Control of Pollution) Act, 1974

    The Air (Prevention and Control of Pollution) Act, 1981

    The Environment (Protection) Act, 1986

    The Wild Life (Protection) Act, 1972

    (b) The Water (Prevention and Control of Pollution) Act, 1972

    The Air (Prevention and Control of Pollution) Act, 1981

    The Environment (Protection) Act, 1986

    The Wild Life (Protection) Act, 1974

    (c) The Water (Prevention and Control of Pollution) Act, 1974

    The Air (Prevention and Control of Pollution) Act, 1986

    The Environment (Protection) Act, 1981

    The Wild Life (Protection) Act, 1972

    (d) The water (Prevention and Control of Pollution) Act, 1981

    The Air (Prevention and Control of Pollution) Act, 1974

    The Environment (Protection) Act, 1986

    The Wild Life (Protection) Act, 1972


    Answer: A 

    Explanation: 

    • Water (Prevention and Control of Pollution) Act, 1974: Aims to prevent and control water pollution by regulating the discharge of pollutants into water bodies, setting water quality standards, and establishing central and state pollution control boards for enforcement. 

    • Air (Prevention and Control of Pollution) Act, 1981: Seeks to prevent and control air pollution by regulating industrial emissions, vehicular pollution, and other sources of air pollutants, with the establishment of central and state pollution control boards. 

    • Environment Protection Act, 1986: Provides a framework for coordinating the activities of various central and state authorities, with the objective of protecting and improving the environment, including the prevention of pollution and conservation of natural resources. 

    • Wildlife Protection Act, 1972: Aims to protect and conserve wildlife by regulating hunting, poaching, and trade in wildlife and their products, establishing protected areas, and promoting the conservation of endangered species. 

    https://cpcb.nic.in/water-pollution/


    https://www.indiacode.nic.in/bitstream/123456789/9462/1/air_act-1981.pdf


    1. Government of Odisha has declared the following Biodiversity Heritage Sites (BHS) for the conservation of Biodiversity in the state.

    Which of the following pairs (Name of the BHS and district) are not correct?

    (a) Gandhamardan Hill Range: Bargarh and Balangir District

    (b) Mandasaru Gorge: Kalahandi District

    (c) Satkosia Gorge: Angul District

    (d) Mahendragiri Hioll range: Gajapati District


    Answer: B 

    Explanation: Mandasaru Gorge ecosystem located in the eastern part of Kandhamal District of Odisha is one of the last kept secret wildernesses of India.Mandasaru Gorge ecosystem located in the eastern part of Kandhamal District of Odisha is one of the last kept secret wildernesses of India. 

    https://www.thehindu.com/news/national/other-states/odisha-declares-gandhamardan-hill-as-biodiversity-heritage-site/article66646261.ece



    1. Acknowledging the years of efforts of tribal people in conserving 130 varieties of indigenous cultivars of rice, Koraput district was declared as a Globally Important Agricultural Heritage System.

    Which one of the following pairs (Organisation: Year) is correct?

    (a) Food and agricultural Organization (FAO) of United Nations: 1982

    (b) Food and Agricultural Organization (FAO) of United Nations: 1992

    (c) Food and Agricultural Organization (FAO) of United Nations: 2002

    (d) Food and Agricultural Organization (FAO) of United Nations: 2012


    Answer: D 

    Explanation: Koraput Traditional Agriculture is one of the Globally Important Agricultural Heritage Systems (GIAHS) since 2012 recognised by the Food and Agriculture Organization of the United Nations. The Globally Important Agricultural Heritage Systems (GIAHS) are agroecosystems inhabited by communities that live in an intricate relationship with their territory. These evolving sites are resilient systems characterised by remarkable agrobiodiversity, traditional knowledge, invaluable cultures and landscapes, sustainably managed by farmers, herders, fisherfolk, and forest people in ways that contribute to their livelihoods and food security. Hence option D is correct.  

    https://vikaspedia.in/aspirational-districts/orissa/koraput/know-your-district/koraput-traditional-agriculture


    1. The Govt. of India had launched “Project Tiger”, the largest species conservation initiative of its kind in the world on 1st April 1973 to promote conservation of the tiger. Nine tiger reserves were established in different states of India in the year 1973.

    Which one of the following combinations does not include the Tiger Reserves established in the year 1973?

    (a) Corbett Tiger Reserve, Similipal Tiger Reserve

    (b) Dholpur-Karauli Tiger Reserve, Satkosia Tiger Reserve

    (c) Manas Tiger Reserve, Sunderbans Tiger Reserve

    (d) Ranthambore Tiger Reserve, Kanha Tiger Reserve

    Answer: B 

    Explanation: In 2023 Dholpur-Karauli Tiger Reserve will become the fifth for big cats in the state following its approval by the National Tiger Conservation Authority (NTCA). The new reserve spans a total area of 1,075 sq km, a buffer zone of 495 sq km and a core area of 580 sq km. The Satkosia Tiger Reserve, located in the Indian state of Odisha, was declared as a tiger reserve in the year 2007. This reserve is situated in the Angul and Nayagarh districts of Odisha and is home to a diverse range of wildlife, including the Bengal tiger Hence Option B is incorrect.  

    https://en.m.wikipedia.org/wiki/Tiger_reserves_of_India


    1. A marine protected area (MPA) is essentially a space in the ocean where human activities are strictly regulated and these places are given special protections for natural or historic marine resources by local, state, territorial, native, regional, or national authorities.

    Which one of the following is not a MPA?

    (a) Periyar in Kerala

    (b) Bhitarkanika in Odisha

    (c) Malvan Marine in Maharashtra

    (d) Coring in Andhra Pradesh


    Answer: A 

    Explanation:


      

    1. The United Nations Conference on Environment and Development (UNCED), also known as the ‘Earth Summit’, was held in Rio de Janeiro, Brazil, from 3-14 June 1992.

    Which of the following combinations of statements about the conference is correct?

    i. This global conference was held on the occasion of 20th anniversary of the first human environment conference held in Stockholm, Sweden

    ii. It focused on the “impact of human socio-economic activities on the environment”

    iii. Representatives from 79 countries participated in the conference

    (a) i and ii

    (b) i and iii

    (c) ii and iii

    (d) i, ii and iii


    Answer: A 

    Explanation: 

    Statement 1 is correct and statement 3 is incorrect: The United Nations Conference on Environment and Development (UNCED), also known as the 'Earth Summit', was held in Rio de Janeiro, Brazil, from 3-14 June 1992. This global conference, held on the occasion of the 20th anniversary of the first Human Environment Conference in Stockholm, Sweden, in 1972, brought together political leaders, diplomats, scientists, representatives of the media and nongovernmental organisations (NGOs) from 179 countries for a massive effort to focus on the impact of human socio-economic activities on the environment. A 'Global Forum' of NGOs was also held in Rio de Janeiro at the same time, bringing together an unprecedented number of NGO representatives, who presented their own vision of the world's future in relation to the environment and socio-economic development. Statement 2 is correct : The Rio de Janeiro conference highlighted how different social, economic and environmental factors are interdependent and evolve together, and how success in one sector requires action in other sectors to be sustained over time. The primary objective of the Rio 'Earth Summit' was to produce a broad agenda and a new blueprint for international action on environmental and development issues that would help guide international cooperation and development policy in the twenty-first century.  

    https://en.wikipedia.org/wiki/Earth_Summit



    1. Microsoft, the recipient of UN Global Action Award, is now committing to remove from the environmental all the carbon the company has emitted either directly or by electrical consumption since it was founded in 1975 by 2050.

    From which year the company has adapted to be carbon neutral?

    (a) 2010

    (b) 2012

    (c) 2014

    (d) 2016


    Answer: B 

    Explanation: Microsoft announced its commitment to becoming carbon neutral in 2012. They set a goal to reduce their carbon emissions to net zero, and they've been working on various initiatives and investments in sustainability to achieve this goal. Hence option B is correct.  

    https://unfccc.int/climate-action/un-global-climate-action-awards/climate-neutral-now/microsoft-carbon-negative-goal#:~:text=Microsoft%20has%20operated%20c


    1. World Environment Day is the United Nations Day for encouraging worldwide awareness and action to protect our environment. In 1972, the UN General Assembly designated 5 June as World Environment Day (WED). The first celebration, under the theme “Only One Earth” took place in 1973. Thereafter every year the day is celebrated with a theme.

    Which of the following pairs (Year and Theme) is not correct?

    i. 2023 : “solutions to Plastic Pollution”

    ii. 2022 : “Only one Earth”

    iii. 2021 : “Ecosystem Restoration”

    iv. 2020 : “Time for Environment”

    (a) i

    (b) i and ii

    (c) ii and iii

    (d) iv


    Answer: A/D 

    Explanation: Option 1 and 4 are incorrect:

    The theme of World Environment Day 2023 is ""BeatPlasticPollution,"" highlighting people's actions toward plastic pollution reduction. The day encourages individuals, governments, and international bodies to prioritise eco-friendly practices for a greener world and the protection of the environment. The theme for World Environment Day 2020 is “Biodiversity” with the slogan “Time for Nature” — a call to action to combat the accelerating species loss and degradation of the natural world. One million plant and animal species risk extinction, largely due to human activities.  

    https://timesofindia.indiatimes.com/education/news/world-environment-day-2023-history-theme-andsignificance/articleshow/100742827.cms#:~:text=This%20year%2C%20the%20theme%20of,the%20protection%20of%20the%20environment.


    General science


    1. One light year is equivalent to roughly how many kilometers?

    (a) 1.5 trillion kilometer

    (b) 4.5 trillion kilometer

    (c) 7.5 trillion kilometer

    (d) 9.5 trillion kilometer

    Answer: D 

    Explanation: One light year is the distance that light travels in one year through a vacuum. It's approximately equal to 9.461 trillion kilometres (about 5.878 trillion miles). Hence option D is correct.  

    https://en.wikipedia.org/wiki/Light-year


    1. Jet fuel used in turbine based aviation engine mainly consists of

    (a) Petrol 

    (b) Diesel 

    (c) Kerosene 

    (d) CNG


    Answer: C 

    Explanation: Jet fuel used in turbine engines for aviation is typically a type of kerosene, known as Jet A or Jet A-1. It's a specific grade of aviation fuel designed for use in jet turbine engines. Jet fuel is chosen for its high energy density and combustion characteristics, which are well-suited for the efficient and high-thrust operation of aircraft engines. Hence option c is correct.

    https://www.ncbi.nlm.nih.gov/books/NBK231234/#


    1. The light emitting diode (LED) is made of

    (a) Si 

    (b) Ge 

    (c) GaN

    (d) Tungsten


    Answer: C 

    Explanation: Light Emitting Diodes (LEDs) are typically made of semiconductor materials. The most commonly used semiconductor materials for LEDs are gallium arsenide (GaAs), gallium phosphide (GaP), and gallium nitride (GaN), depending on the desired wavelength or color of light emission. Hence option c is correct.

    https://www.britannica.com/technology/LED


    1. Meniscus rise in glass thermometer is due to the change in

    (a) Density 

    (b) Mass 

    (c) Heat capacity

    (d) Thermal conductivity


    Answer: A 

    Explanation: When the temperature increases, the liquid (usually mercury or alcohol) in the thermometer expands, causing its density to decrease. This decrease in density leads to the meniscus rising in the capillary tube. Hence option A is correct. 

    https://www.acs.org/middleschoolchemistry/lessonplans/chapter1/lesson3.html#:~:text=The%20way%20a%20thermometer%20works,in%20movement%20up%20the%20thermometer.


    1. A glass of (1) tap water, (2) distilled water and (3) sea water is cooled to its solidification. The order in which they will solidify first is

    (a) tap water, distilled water, sea water

    (b) distilled water, sea water, tap water

    (c) sea water, tap water, distilled water

    (d) distilled water, tap water, sea water


    Answer: D 

    Explanation: Distilled Water: Distilled water is pure and has no impurities or salts, which means it can freeze at 0°C (32°F), the normal freezing point for pure water. Tap Water: Tap water, which typically contains various dissolved minerals and impurities, will freeze after distilled water. Sea Water: Sea water contains a significant amount of dissolved salts and impurities. As a result, it has a lower freezing point than pure water. Sea water freezes at a lower temperature, typically around -2°C (28°F) to -1.8°C (28.8°F), due to the presence of salts. So distilled water will solidify first, followed by tap water (if it's relatively pure), and sea water will solidify last due to its lower freezing point caused by the dissolved salts. Hence option D is correct. 

    Post a Comment

    0 Comments